95
INSIGHTSIAS SIMPLYFYING IAS EXAM PREPARATION INSTA Tests 41 to 44 (GS) www.insightsonindia.com prelims.insightsonindia.com | mains.insightsonindia.com Telegram: insightsIAStips | FB: insightsonindia | TW: vinaygb | YT: INSIGHTS IAS BENGALURU | DELHI | HYDERABAD INSTA 75 Days REVISION PLAN UPSC Prelims 2020 Copyright © by Insights IAS All rights are reserved. No part of this document may be reproduced, stored in a retrieval system or transmitted in any form or by any means, electronic, mechanical, photocopying, recording or otherwise, without prior permission of Insights IAS. KEY & EXPLANATIONS

SIMPLYFYING IAS EXAM PREPARATIONDAY – 41 (InstaTest-41) 1. Consider the following statements regarding Janaushadhi Sugam app 1. It has been launched by Ministry of Health and Family

  • Upload
    others

  • View
    1

  • Download
    0

Embed Size (px)

Citation preview

Page 1: SIMPLYFYING IAS EXAM PREPARATIONDAY – 41 (InstaTest-41) 1. Consider the following statements regarding Janaushadhi Sugam app 1. It has been launched by Ministry of Health and Family

INSIGHTSIAS SIMPLYFYING IAS EXAM PREPARATION

INSTA Tests

41 to 44 (GS)

www.insightsonindia.com

prelims.insightsonindia.com | mains.insightsonindia.com

Telegram: insightsIAStips | FB: insightsonindia | TW: vinaygb | YT: INSIGHTS IAS

BENGALURU | DELHI | HYDERABAD

INSTA 75 Days REVISION PLAN UPSC Prelims 2020

Copyright © by Insights IAS All rights are reserved. No part of this document may be reproduced, stored in a retrieval system or transmitted in any form or by any means, electronic, mechanical, photocopying, recording or otherwise, without prior permission of Insights IAS.

KEY & EXPLANATIONS

Page 2: SIMPLYFYING IAS EXAM PREPARATIONDAY – 41 (InstaTest-41) 1. Consider the following statements regarding Janaushadhi Sugam app 1. It has been launched by Ministry of Health and Family

Insta 75 Days Revision Plan for UPSC Civil Services

Prelims – 2020

This document is the compilation of 100 questions that are part of InsightsIAS

famous INSTA REVISION initiative for UPSC civil services Preliminary examination

– 2020 (which has become most anticipated annual affair by lakhs of IAS aspirants

across the country). These questions are carefully framed so as to give aspirants tough

challenge to test their knowledge and at the same time improve skills such as

intelligent guessing, elimination, reasoning, deduction etc – which are much needed

to sail through tough Civil Services Preliminary Examination conducted by UPSC.

These questions are based on this INSTA Revision Plan which is posted on our

website (www.insightsonindia.com). Every year thousands of candidates follow our

revision timetable – which is made for SERIOUS aspirants who would like to intensively

revise everything that’s important before the exam.

Those who would like to take up more tests for even better preparation, can

enroll to Insights IAS Prelims Mock Test Series – 2020

(https://prelims.insightsonindia.com). Every year toppers solve our tests and sail

through UPSC civil services exam. Your support through purchase of our tests will help

us provide FREE content on our website seamlessly.

Wish you all the best!

Team InsightsIAS

Page 3: SIMPLYFYING IAS EXAM PREPARATIONDAY – 41 (InstaTest-41) 1. Consider the following statements regarding Janaushadhi Sugam app 1. It has been launched by Ministry of Health and Family

INSTA 75 Days REVISION PLAN for Prelims 2020 - InstaTests

www.insightsonindia.com 1 Insights IAS

DAY – 41 (InstaTest-41)

1. Consider the following statements regarding Janaushadhi Sugam app

1. It has been launched by Ministry of Health and Family Welfare.

2. It will help to get online medical consultation by authorized Doctors.

Which of the statements given above is/are correct?

(a) 1 only

(b) 2 only

(c) Both 1 and 2

(d) Neither 1 nor 2

Solution: D

Janaushadhi Sugam

• Union Ministry for Chemicals and Fertilizers has launched a mobile application

“Janaushadhi Sugam”.

• The application aims to enable people to search Janaushadhi generic medicines and

the stores at the tip of their fingers.

• It will also help analyse product comparison of Generic vs Branded medicine in form

of MRP & overall Savings.

• Janaushadhi Sugam mobile application would have user-friendly options like- to locate

nearby Janaushadhikendra, direction guidance for location of the Janaushadhikendra

through Google Map, search Janaushadhi generic medicines, analyse product

comparison of Generic vs Branded medicine in form of MRP & overall Savings, etc.

• It can be downloaded free of cost by the user from Google Play Store and Apple Store.

2. Consider the following statements regarding Ordinance-making power of the president

1. President can promulgate an ordinance when both the Houses of Parliament are

not in session.

2. An ordinance unlike any other legislation, cannot be retrospective.

3. An ordinance can alter or amend a tax law.

Which of the statements given above is/are correct?

(a) 1 only

(b) 2 and 3 only

Page 4: SIMPLYFYING IAS EXAM PREPARATIONDAY – 41 (InstaTest-41) 1. Consider the following statements regarding Janaushadhi Sugam app 1. It has been launched by Ministry of Health and Family

INSTA 75 Days REVISION PLAN for Prelims 2020 - InstaTests

www.insightsonindia.com 2 Insights IAS

(c) 1 and 3 only

(d) 1, 2 and 3

Solution: C

Ordinance-making power of the president

• Article 123 of the Constitution empowers the President to promulgate ordinances

during the recess of Parliament.

• He can promulgate an ordinance only when both the Houses of Parliament are not in

session or when either of the two Houses of Parliament is not in session.

• Every ordinance issued by the President during the recess of Parliament must be laid

before both the Houses of Parliament when it reassembles.

• If an ordinance is allowed to lapse without being placed before Parliament, then the

acts done and completed under it, before it ceases to operate, remain fully valid and

effective.

• An ordinance like any other legislation, can be retrospective, that is, it may come into

force from a back date. It may modify or repeal any act of Parliament or another

ordinance. It can alter or amend a tax law also.

3. Consider the following statements regarding Electoral College of Vice-President

1. It consists of elected members of the Parliament

2. It consists of nominated members of the Parliament

3. It consists of the members of the state legislative assemblies

Which of the statements given above is/are correct?

(a) 1 and 2 only

(b) 2 and 3 only

(c) 1 only

(d) 1, 2 and 3

Solution: A

The Vice-President, like the president, is elected not directly by the people but by the

method of indirect election. He is elected by the members of an electoral college consisting

of the members of both Houses of Parliament. Thus, this electoral college is different from

the electoral college for the election of the President in the following two respects:

Page 5: SIMPLYFYING IAS EXAM PREPARATIONDAY – 41 (InstaTest-41) 1. Consider the following statements regarding Janaushadhi Sugam app 1. It has been launched by Ministry of Health and Family

INSTA 75 Days REVISION PLAN for Prelims 2020 - InstaTests

www.insightsonindia.com 3 Insights IAS

• It consists of both elected and nominated members of the Parliament (in the case of

president, only elected members).

• It does not include the members of the state legislative assemblies (in the case of

President, the elected members of the state legislative assemblies are included).

• The Vice-President’s election, like that of the President’s election, is held in

accordance with the system of proportional representation by means of the single

transferable vote and the voting is by secret ballot.

• All doubts and disputes in connection with election of the Vice-President are inquired

into and decided by the Supreme Court whose decision is final.

4. Consider the following statements regarding North East Rural Livelihood Project

(NERLP)

1. It has been implemented in all the districts of North-eastern states.

2. The project aims to improve rural livelihoods especially that of women,

unemployed youth and the most disadvantaged in north eastern states.

3. It is Asian Development Bank (ADB) aided project.

Which of the statements given above is/are correct?

(a) 1 and 2 only

(b) 2 and 3 only

(c) 2 only

(d) 1 and 3 only

Solution: C

North East Rural Livelihood Project (NERLP)

• A study finds that North East Rural Livelihood Project (NERLP) improves livelihoods

of 300,000 households in 11 districts of Mizoram, Nagaland, Tripura and Sikkim. Under

this project, the Skills development and placement has trained 10462 boys and girls in

various job skills and a total of 5494 of them are employed today.

About NERLP:

• It is a World Bank aided, multi-state livelihood project under the Ministry of

Development of North Eastern Region (DoNER), launched in 2012.

• Implemented in 11 districts of Mizoram, Nagaland, Tripura and Sikkim.

• Aim: to improve rural livelihoods especially that of women, unemployed youth and

the most disadvantaged, in four North Eastern States (Mizoram, Nagaland, Tripura and

Sikkim)

Page 6: SIMPLYFYING IAS EXAM PREPARATIONDAY – 41 (InstaTest-41) 1. Consider the following statements regarding Janaushadhi Sugam app 1. It has been launched by Ministry of Health and Family

INSTA 75 Days REVISION PLAN for Prelims 2020 - InstaTests

www.insightsonindia.com 4 Insights IAS

• The project has focussed on five development strategies, namely, social

empowerment, economic empowerment, partnership development, project

management and livelihood & value chain developments.

5. Consider the following statements regarding the appointment of the Prime Minister

1. The Constitution does not contain any specific procedure for the selection and

appointment of the Prime Minister.

2. The Constitution specifies the term of Prime Minister and he holds the office for

a term of five years.

Which of the statements given above is/are correct?

(a) 1 only

(b) 2 only

(c) Both 1 and 2

(d) Neither 1 nor 2

Solution: A

Appointment of The Prime Minister

• The Constitution does not contain any specific procedure for the selection and

appointment of the Prime Minister. Article 75 says only that the Prime Minister shall

be appointed by the president.

• However, this does not imply that the president is free to appoint any one as the Prime

Minister. In accordance with the conventions of the parliamentary system of

government, the President has to appoint the leader of the majority party in the Lok

Sabha as the Prime Minister.

• The term of the Prime Minister is not fixed and he holds office during the pleasure

of the president. However, this does not mean that the president can dismiss the

Prime Minister at any time. So long as the Prime Minister enjoys the majority support

in the Lok Sabha, he cannot be dismissed by the President.

6. Consider the following statements regarding Nature of advice by ministers

1. The 42nd and 44th Constitutional Amendment Acts have made the advice binding

on the President.

2. After the dissolution of the Lok Sabha, the council of ministers does not cease to

hold office.

Which of the statements given above is/are correct?

Page 7: SIMPLYFYING IAS EXAM PREPARATIONDAY – 41 (InstaTest-41) 1. Consider the following statements regarding Janaushadhi Sugam app 1. It has been launched by Ministry of Health and Family

INSTA 75 Days REVISION PLAN for Prelims 2020 - InstaTests

www.insightsonindia.com 5 Insights IAS

(a) 1 only

(b) 2 only

(c) Both 1 and 2

(d) Neither 1 nor 2

Solution: C

• The 42nd and 44th Constitutional Amendment Acts have made the advice binding on

the President. Further, the nature of advice tendered by ministers to the President

cannot be enquired by any court. This provision emphasizes the intimate and the

confidential relationship between the President and the ministers.

• In 1971, the Supreme Court held that ‘even after the dissolution of the Lok Sabha,

the council of ministers does not cease to hold office. Article 74 is mandatory and,

therefore, the president cannot exercise the executive power without the aid and

advise of the council of ministers. Any exercise of executive power without the aid

and advice will be unconstitutional as being violative of Article 74’. Again in 1974, the

court held that ‘wherever the Constitution requires the satisfaction of the President,

the satisfaction is not the personal satisfaction of the President but it is the satisfaction

of the council of ministers with whose aid and on whose advice the President exercises

his powers and functions’.

7. Consider the following statements regarding oxytocin

1. It is naturally secreted by the pituitary glands of mammals

2. It acts both as a hormone and as a brain neurotransmitter.

3. It can be administered to humans as an injection or a nasal solution.

Which of the statements given above is/are correct?

(a) 1 and 2 only

(b) 3 only

(c) 2 and 3 only

(d) 1, 2 and 3

Solution: D

Page 8: SIMPLYFYING IAS EXAM PREPARATIONDAY – 41 (InstaTest-41) 1. Consider the following statements regarding Janaushadhi Sugam app 1. It has been launched by Ministry of Health and Family

INSTA 75 Days REVISION PLAN for Prelims 2020 - InstaTests

www.insightsonindia.com 6 Insights IAS

Oxytocin:

• The Delhi High Court set aside the government decision to ban private firms from

producing and selling oxytocin, a drug used for inducing labour contractions and

controlling bleeding.

• Oxytocin has also been dubbed the hug hormone, cuddle chemical, moral molecule,

and the bliss hormone due to its effects on behavior, including its role in love and in

female reproductive biological functions in reproduction.

• Oxytocin is a hormone that is made in the brain, in the hypothalamus. It is transported

to, and secreted by, the pituitary gland, which is located at the base of the brain.

• It acts both as a hormone and as a brain neurotransmitter.

• The release of oxytocin by the pituitary gland acts to regulate two female reproductive

functions: Childbirth and Breast-feeding.

• Oxytocin can be administered to humans as an injection or a nasal solution.

• It is chemically synthesized and sold by pharmaceutical companies across the world.

8. Which of the following Cabinet Committees are chaired by Prime Minister

1. Cabinet Committee on Parliamentary Affairs

2. Cabinet Committee on Political Affairs

3. Cabinet Committee on Investment and Growth

4. Cabinet Committee on Employment and Skill Development

Select the correct answer using the code given below:

(a) 1 and 2 only

(b) 2 and 3 only

(c) 2, 3 and 4 only

(d) 1, 2 and 4 only

Solution: C

• The Union government has released the composition of eight Cabinet Committees,

including two new ones — one on Investment, the other on Employment and Skill

Development.

What are these Cabinet Committees for?

• Government of India Transaction of Business Rules, 1961 emerging out of Article

77(3) of the Constitution states: “The President shall make rules for the more

convenient transaction of the business of the Government of India, and for the

allocation among Ministers of the said business.”

Page 9: SIMPLYFYING IAS EXAM PREPARATIONDAY – 41 (InstaTest-41) 1. Consider the following statements regarding Janaushadhi Sugam app 1. It has been launched by Ministry of Health and Family

INSTA 75 Days REVISION PLAN for Prelims 2020 - InstaTests

www.insightsonindia.com 7 Insights IAS

• The Rules mandate the minister-in-charge of a department (ministry) to dispose of “all

business allotted to a department under” him or her. However, “when the subject of

a case concerns more than one department”, no decision can be taken “until all such

departments have concurred, or, failing such concurrence, a decision thereon has

been taken by or under the authority of the Cabinet”.

Who constitutes and assigns functions to these committees?

• The Prime Minister constitutes Standing Committees of the Cabinet and sets out the

specific functions assigned to them. He can add or reduce the number of committees.

• Ad hoc committees of ministers, including Groups of Ministers, may be appointed by

the Cabinet or by the Prime Minister for specific matters.

At present (2019), the following 8 Cabinet Committees are functional:

1. Cabinet Committee on Political Affairs

2. Cabinet Committee on Economic Affairs

3. Appointments Committee of the Cabinet

4. Cabinet Committee on Security

5. Cabinet Committee on Parliamentary Affairs

6. Cabinet Committee on Accommodation

7. Cabinet Committee on Investment and Growth

8. Cabinet Committee on Employment and Skill Development

Except Cabinet Committee on Accommodation and Cabinet Committee on Parliamentary

Affairs all others are chaired by PM.

Cabinet Committee on Accommodation: Shri Amit Shah, Minister of Home Affairs

Cabinet Committee on Parliamentary Affairs: Shri Raj Nath Singh, Minister of Defence.

9. Consider the following statements regarding Duration of Rajya Sabha

1. The Constitution has fixed the term of office of members of the Rajya Sabha for 6

years.

2. The Rajya Sabha is a continuing chamber, that is, it is a permanent body and not

subject to dissolution.

Which of the statements given above is/are correct?

(a) 1 only

(b) 2 only

(c) Both 1 and 2

(d) Neither 1 nor 2

Solution: B

Page 10: SIMPLYFYING IAS EXAM PREPARATIONDAY – 41 (InstaTest-41) 1. Consider the following statements regarding Janaushadhi Sugam app 1. It has been launched by Ministry of Health and Family

INSTA 75 Days REVISION PLAN for Prelims 2020 - InstaTests

www.insightsonindia.com 8 Insights IAS

Duration of Rajya Sabha

• The Rajya Sabha (first constituted in 1952) is a continuing chamber, that is, it is a

permanent body and not subject to dissolution. However, one-third of its members

retire every second year. Their seats are filled up by fresh elections and presidential

nominations at the beginning of every third year. The retiring members are eligible for

re-election and re-nomination any number of times.

• The Constitution has not fixed the term of office of members of the Rajya Sabha and

left it to the Parliament. Accordingly, the Parliament in the Representation of the

People Act (1951) provided that the term of office of a member of the Rajya Sabha

shall be six years. The act also empowered the president of India to curtail the term of

members chosen in the first Rajya Sabha. In the first batch, it was decided by lottery

as to who should retire. Further, the act also authorized the President to make

provisions to govern the order of retirement of the members of the Rajya Sabha.

10. Consider the following statements regarding National Productivity Council (NPC)

1. It is established by the Ministry of Labour and Employment

2. It is a tri-partite organization with equal representation from government,

employers and workers’ organizations.

3. It aims to stimulate and promote productivity and quality consciousness across all

sectors in the country.

Which of the statements given above is/are correct?

(a) 1 and 3 only

(b) 1 and 2 only

(c) 2 and 3 only

(d) 1, 2 and 3

Solution: C

National Productivity Council (NPC)

• NPC is a national level autonomous organization under Department of Industrial

Policy & Promotion, Ministry of Commerce & Industry to promote productivity

culture in India.

• It was established as a registered society on 12th February 1958 by the Government

with aim to stimulate and promote productivity and quality consciousness across all

sectors in the country.

• It is a tri-partite organization with equal representation from government, employers

and workers’ organizations.

Page 11: SIMPLYFYING IAS EXAM PREPARATIONDAY – 41 (InstaTest-41) 1. Consider the following statements regarding Janaushadhi Sugam app 1. It has been launched by Ministry of Health and Family

INSTA 75 Days REVISION PLAN for Prelims 2020 - InstaTests

www.insightsonindia.com 9 Insights IAS

• NPC is a constituent of the Tokyo-based Asian Productivity Organisation (APO), an

Intergovernmental Body, of which the Government of India is a founder member.

The objectives of NPC are as follows:

• To create and develop productivity consciousness in the country.

• To make arrangements for the training of managers at every level of management.

• To make arrangements for the services of experts on the requisitions of local

productivity councils.

• To undertake research with regard to various production processes.

• To send delegations to developed countries to collect and study information with

regard to increased productivity.

• To make arrangements for training of personnel with regard to productivity methods

and techniques in other countries.

• To invite experts on productivity from foreign countries and to make use of their

knowledge and services.

• To import various productivity services with a view to having maximum utilisation of

available resources, viz; men, money, materials and machines.

• To ensure higher and better living standards to the people of the country.

11. Consider the following statements regarding Whip

1. The office of whip is mentioned neither in the Constitution of India nor in the

Rules of the House.

2. It is based on the conventions of the parliamentary government.

3. Every political party, whether ruling or Opposition has its own whip in the

Parliament.

Which of the statements given above is/are correct?

(a) 1 and 2 only

(b) 2 and 3 only

(c) 1 and 3 only

(d) 1, 2 and 3

Solution: D

Whip

• Though the offices of the leader of the House and the leader of the Opposition are

not mentioned in the Constitution of India, they are mentioned in the Rules of the

House and Parliamentary Statute respectively. The office of ‘whip’, on the other

Page 12: SIMPLYFYING IAS EXAM PREPARATIONDAY – 41 (InstaTest-41) 1. Consider the following statements regarding Janaushadhi Sugam app 1. It has been launched by Ministry of Health and Family

INSTA 75 Days REVISION PLAN for Prelims 2020 - InstaTests

www.insightsonindia.com 10 Insights IAS

hand, is mentioned neither in the Constitution of India nor in the Rules of the House

nor in a Parliamentary Statute. It is based on the conventions of the parliamentary

government.

• Every political party, whether ruling or Opposition has its own whip in the Parliament.

He is appointed by the political party to serve as an assistant floor leader. He is charged

with the responsibility of ensuring the attendance of his party members in large

numbers and securing their support in favor of or against a particular issue. He

regulates and monitors their behavior in the Parliament. The members are supposed

to follow the directives given by the whip. Otherwise, disciplinary action can be taken.

12. Consider the following statements regarding Closure Motion

1. Simple Closure is one when a member moves that the matter having been

sufficiently discussed be now put to vote.

2. Guillotine Closure is one when the undiscussed clauses of a bill or a resolution are

also put to vote along with the discussed ones.

3. Under the Closure by Compartments, only important clauses are taken up for

debate and voting and the intervening clauses are skipped.

Which of the statements given above is/are correct?

(a) 1 and 2 only

(b) 2 only

(c) 1 and 3 only

(d) 1, 2 and 3

Solution: A

Closure Motion

It is a motion moved by a member to cut short the debate on a matter before the House. If

the motion is approved by the House, debate is stopped forthwith and the matter is put to

vote. There are four kinds of closure motions:

(a) Simple Closure: It is one when a member moves that the ‘matter having been sufficiently

discussed be now put to vote’.

(b) Closure by Compartments: In this case, the clauses of a bill or a lengthy resolution are

grouped into parts before the commencement of the debate. The debate covers the part

as a whole and the entire part is put to vote.

(c) Kangaroo Closure: Under this type, only important clauses are taken up for debate and

voting and the intervening clauses are skipped over and taken as passed.

Page 13: SIMPLYFYING IAS EXAM PREPARATIONDAY – 41 (InstaTest-41) 1. Consider the following statements regarding Janaushadhi Sugam app 1. It has been launched by Ministry of Health and Family

INSTA 75 Days REVISION PLAN for Prelims 2020 - InstaTests

www.insightsonindia.com 11 Insights IAS

(d) Guillotine Closure: It is one when the undiscussed clauses of a bill or a resolution are also

put to vote along with the discussed ones due to want of time (as the time allotted for

the discussion is over).

13. Which one of the following does not share border with Panama?

(a) Costa Rica

(b) Pacific Ocean

(c) Colombia

(d) Venezuela

Solution: D

14. Consider the following statements regarding Adjournment Motion

1. It is introduced in the Parliament to draw attention of the House to a definite

matter of urgent public importance.

2. The discussion on an adjournment motion should last for less than two hours and

thirty minutes.

3. It can be introduced in both the houses of the parliament.

Which of the statements given above is/are correct?

(a) 1 only

(b) 1 and 2 only

Page 14: SIMPLYFYING IAS EXAM PREPARATIONDAY – 41 (InstaTest-41) 1. Consider the following statements regarding Janaushadhi Sugam app 1. It has been launched by Ministry of Health and Family

INSTA 75 Days REVISION PLAN for Prelims 2020 - InstaTests

www.insightsonindia.com 12 Insights IAS

(c) 2 and 3 only

(d) 1, 2 and 3

Solution: A

Adjournment Motion

• It is introduced in the Parliament to draw attention of the House to a definite matter

of urgent public importance, and needs the support of 50 members to be admitted.

As it interrupts the normal business of the House, it is regarded as an extraordinary

device. It involves an element of censure against the government and hence Rajya

Sabha is not permitted to make use of this device.

The discussion on an adjournment motion should last for not less than two hours and thirty

minutes. The right to move a motion for an adjournment of the business of the House is

subject to the following restrictions:

• It should raise a matter which is definite, factual, urgent and of public importance;

• It should not cover more than one matter;

• It should be restricted to a specific matter of recent occurrence and should not be

framed in general terms;

• It should not raise a question of privilege;

• It should not revive discussion on a matter that has been discussed in the same

session;

• It should not deal with any matter that is under adjudication by court; and

• It should not raise any question that can be raised on a distinct motion.

15. Consider the following statements regarding Public Accounts Committee

1. This committee was set up under the provisions of the Government of India Act

of 1935.

2. By convention, the chairman of the committee is selected invariably from the

Opposition.

3. The CAG acts as a guide, friend and philosopher of the committee.

Which of the statements given above is/are correct?

(a) 2 only

(b) 2 and 3 only

(c) 3 only

(d) 1, 2 and 3

Page 15: SIMPLYFYING IAS EXAM PREPARATIONDAY – 41 (InstaTest-41) 1. Consider the following statements regarding Janaushadhi Sugam app 1. It has been launched by Ministry of Health and Family

INSTA 75 Days REVISION PLAN for Prelims 2020 - InstaTests

www.insightsonindia.com 13 Insights IAS

Solution: B

Public Accounts Committee

• This committee was set up first in 1921 under the provisions of the Government of

India Act of 1919 and has since been in existence. At present, it consists of 22

members (15 from the Lok Sabha and 7 from the Rajya Sabha). The members are

elected by the Parliament every year from amongst its members according to the

principle of proportional representation by means of the single transferable vote.

Thus, all parties get due representation in it.

• The term of office of the members is one year. A minister cannot be elected as a

member of the committee. The chairman of the committee is appointed from

amongst its members by the Speaker. Until 1966 – ‘67, the chairman of the committee

belonged to the ruling party. However, since 1967 a convention has developed

whereby the chairman of the committee is selected invariably from the Opposition.

• The committee is assisted by the CAG. In fact, the CAG acts as a guide, friend and

philosopher of the committee.

16. Consider the following statements regarding Index of Eight Core Industries

1. It is compiled and released by Central Statistics Office.

2. Electricity has the highest weightage in the Index.

3. The eight core industries comprise 60% of the weight of items included in the

Index of Industrial Production (IIP).

Which of the statements given above is/are correct?

(a) 1 and 3 only

(b) 2 only

(c) 1 only

(d) None

Solution: D

Index of Eight Core Industries

It is the monthly index of production volume which measures collective and individual

performance of eight core industries viz.

1. Petroleum RefInery Products (weight: 28.04%)

2. Electricity (19.85%)

3. Steel (17.92%)

Page 16: SIMPLYFYING IAS EXAM PREPARATIONDAY – 41 (InstaTest-41) 1. Consider the following statements regarding Janaushadhi Sugam app 1. It has been launched by Ministry of Health and Family

INSTA 75 Days REVISION PLAN for Prelims 2020 - InstaTests

www.insightsonindia.com 14 Insights IAS

4. Coal (10.33%)

5. Crude Oil (8.98%)

6. Natural Gas (6.88%)

7. Cement. (5.37%)

8. Fertilizers (2.63%)

It is compiled and released by Office of Economic Adviser (OEA), Department for Promotion

of Industry and Internal Trade, Ministry of Commerce & Industry.

The Eight Core Industries comprise 40.27 per cent of the weight of items included in the Index

of Industrial Production (lIP).

The base year for the Index currently is 2011-12.

17. Consider the following statements regarding composition of Parliamentary Forum

1. The Speaker of Lok Sabha is the ex-officio President of all the Forums.

2. The duration of the office of members of the forum is one year.

Which of the statements given above is/are correct?

(a) 1 only

(b) 2 only

(c) Both 1 and 2

(d) Neither 1 nor 2

Solution: D

The first Parliamentary Forum on Water Conservation and Management was constituted in

the year 2005.1 Subsequently, seven more Parliamentary forums were constituted.

COMPOSITION OF THE FORUMS

• The Speaker of Lok Sabha is the ex-officio President of all the Forums except the

Parliamentary Forum on Population and Public Health wherein the Chairman of

Rajya Sabha is the ex-officio President and the Speaker is the ex-officio Co-President.

The Deputy Chairman of Rajya Sabha, the Deputy Speaker of Lok Sabha, the concerned

Ministers and the Chairman of Departmentally-Related Standing Committees are the

ex-officio Vice-presidents of the respective Forums.

• Each Forum consists of not more than 31 members (excluding the President, Co-

President and Vice-Presidents) out of whom not more than 21 are from the Lok Sabha

and not more than 10 are from the Rajya Sabha.

• Members (other than the President, Co-President and Vice-Presidents) of these

forums are nominated by the Speaker/Chairman from amongst the leaders of various

Page 17: SIMPLYFYING IAS EXAM PREPARATIONDAY – 41 (InstaTest-41) 1. Consider the following statements regarding Janaushadhi Sugam app 1. It has been launched by Ministry of Health and Family

INSTA 75 Days REVISION PLAN for Prelims 2020 - InstaTests

www.insightsonindia.com 15 Insights IAS

political parties/groups or their nominees, who have special knowledge/keen interest

in the subject.

• The duration of the office of members of the forum is coterminous with their

membership in the respective Houses. A member may also resign from the forum by

writing to the Speaker/Chairman.

18. Consider the following statements regarding Ad hoc Judge of supreme court

1. The President can appoint a judge of a High Court as an ad hoc judge of the

Supreme Court for a temporary period.

2. The judge so appointed should be qualified for appointment as a judge of the

Supreme Court.

Which of the statements given above is/are correct?

(a) 1 only

(b) 2 only

(c) Both 1 and 2

(d) Neither 1 nor 2

Solution: B

Ad hoc Judge

• When there is a lack of quorum of the permanent judges to hold or continue any

session of the Supreme Court, the Chief Justice of India can appoint a judge of a High

Court as an ad hoc judge of the Supreme Court for a temporary period. He can do so

only after consultation with the chief justice of the HighCourt concerned and with the

previous consent of the president. The judge so appointed should be qualified for

appointment as a judge of the Supreme Court. It is the duty of the judge so appointed

to attend the sittings of the Supreme Court, in priority to other duties of his office.

While so attending, he enjoys all the jurisdiction, powers and privileges (and

discharges the duties) of a judge of the Supreme Court.

Retired Judge

• At any time, the chief justice of India can request a retired judge of the Supreme Court

or a retired judge of a high court (who is duly qualified for appointment as a judge of

the Supreme Court) to act as a judge of the Supreme Court for a temporary period. He

can do so only with the previous consent of the president and also of the person to be

so appointed. Such a judge is entitled to such allowances as the president may

determine. He will also enjoy all the jurisdiction, powers and privileges of a judge of

Page 18: SIMPLYFYING IAS EXAM PREPARATIONDAY – 41 (InstaTest-41) 1. Consider the following statements regarding Janaushadhi Sugam app 1. It has been launched by Ministry of Health and Family

INSTA 75 Days REVISION PLAN for Prelims 2020 - InstaTests

www.insightsonindia.com 16 Insights IAS

Supreme Court. But he will not otherwise be deemed to be a judge of the Supreme

Court.

19. Consider the following statements regarding Equalization Levy

1. It was introduced to tax the income gained by foreign e-commerce companies

through digital transactions from India.

2. The equalization levy was introduced to give effect to recommendations of the

OECD on Base Erosion and Profit Shifting Action Plan.

Which of the statements given above is/are correct?

(a) 1 only

(b) 2 only

(c) Both 1 and 2

(d) Neither 1 nor 2

Solution: C

About Equalization levy

• Equalization levy was introduced to tax the income gained by foreign e-commerce

companies through digital transactions from India. It is aimed at taxing business to

business transactions.

• The equalization levy was introduced vide Budget 2016 to give effect to the

recommendations of the OECD on Base Erosion and Profit Shifting Action Plan.

https://economictimes.indiatimes.com/tech/internet/big-tech-takes-equalisation-levy-row-

to-us-govt/articleshow/74922783.cms?from=mdr

20. Consider the following statements regarding Discretionary powers of the Governor and

president

1. While the Constitution envisages the possibility of the governor acting at times in

his discretion, no such possibility has been envisaged for the President.

2. After the 42nd Constitutional Amendment (1976), ministerial advice has been

made binding on the President, but no such provision has been made with respect

to the governor.

Which of the statements given above is/are correct?

(a) 1 only

(b) 2 only

Page 19: SIMPLYFYING IAS EXAM PREPARATIONDAY – 41 (InstaTest-41) 1. Consider the following statements regarding Janaushadhi Sugam app 1. It has been launched by Ministry of Health and Family

INSTA 75 Days REVISION PLAN for Prelims 2020 - InstaTests

www.insightsonindia.com 17 Insights IAS

(c) Both 1 and 2

(d) Neither 1 nor 2

Solution: C

Constitutional position of the governor differs from that of the president in the following two

respects:

• While the Constitution envisages the possibility of the governor acting at times in his

discretion, no such possibility has been envisaged for the President.

• After the 42nd Constitutional Amendment (1976), ministerial advice has been made

binding on the President, but no such provision has been made with respect to the

governor.

The Constitution makes it clear that if any question arises whether a matter falls within the

governor’s discretion or not, the decision of the governor is final and the validity of anything

done by him cannot be called in question on the ground that he ought or ought not to have

acted in his discretion. The governor has constitutional discretion in the following cases:

1. Reservation of a bill for the consideration of the President.

2. Recommendation for the imposition of the President’s Rule in the state.

3. While exercising his functions as the administrator of an adjoining union territory (in

case of additional charge).

4. Determining the amount payable by the Government of Assam, Meghalaya, Tripura

and Mizoram to an autonomous Tribal District Council as royalty accruing from

licenses for mineral exploration.

5. Seeking information from the chief minister with regard to the administrative and

legislative matters of the state.

21. Consider the following statements regarding National Board of Wildlife

1. It is a statutory board constituted in 2003 under the Wild Life (Protection) Act,

1972.

2. No alteration of the boundaries of a National Park is allowed except on a

recommendation of the National Board of Wildlife.

3. No alteration or de-notification of Tiger Reserves is allowed without the approval

of the National Board for Wildlife.

Which of the statements given above is/are correct?

(a) 1 and 3 only

(b) 2 only

(c) 2 and 3 only

Page 20: SIMPLYFYING IAS EXAM PREPARATIONDAY – 41 (InstaTest-41) 1. Consider the following statements regarding Janaushadhi Sugam app 1. It has been launched by Ministry of Health and Family

INSTA 75 Days REVISION PLAN for Prelims 2020 - InstaTests

www.insightsonindia.com 18 Insights IAS

(d) 1, 2 and 3

Solution: D

National Board for Wildlife (NBWL) is a statutory Board constituted in 2003 under the Wild

Life (Protection) Act, 1972.

• It is the apex body tasked with according permissions for forest land diversions for

industrial development. The NBWL is a 47 member body, chaired by the Prime

Minister, of which 19 members are ex-officio members.

Functions

• No construction of commercial lodges, hotel etc. shall be undertaken except with the

prior approval of the National Board.

• No alteration of the boundaries of a National Park except on a recommendation of the

National Board.

• No destruction, removal of wildlife or forest produce from a National Park or diversion

of habitat unless

• State Government in consultation with the National Board authorizes the issue of such

permit.

• Ensure Tiger Reserves and areas linking one protected area with another are not

diverted for ecologically unsustainable uses except in public interest and with the

approval of the National Board.

• No alteration or de-notification of Tiger Reserves without the approval of the National

Board for Wildlife.

22. Consider the following statements regarding Composition of Legislative Council

1. The maximum strength of the council is fixed at one-third of the total strength of

the assembly and the minimum strength is fixed at 40.

2. The actual strength of a Council is fixed by Parliament.

3. The Parliament is authorized to modify scheme of composition of a legislative

council.

Which of the statements given above is/are correct?

(a) 1 and 2 only

(b) 2 and 3 only

(c) 1 and 3 only

(d) 1, 2 and 3

Page 21: SIMPLYFYING IAS EXAM PREPARATIONDAY – 41 (InstaTest-41) 1. Consider the following statements regarding Janaushadhi Sugam app 1. It has been launched by Ministry of Health and Family

INSTA 75 Days REVISION PLAN for Prelims 2020 - InstaTests

www.insightsonindia.com 19 Insights IAS

Solution: D

Composition of Council Strength

• Unlike the members of the legislative assembly, the members of the legislative council

are indirectly elected. The maximum strength of the council is fixed at one-third of

the total strength of the assembly and the minimum strength is fixed at 40. It means

that the size of the council depends on the size of the assembly of the concerned state.

• This is done to ensure the predominance of the directly elected House (assembly) in

the legislative affairs of the state. Though the Constitution has fixed the maximum and

the minimum limits, the actual strength of a Council is fixed by Parliament.

Manner of Election

Of the total number of members of a legislative council:

• 1/3 are elected by the members of local bodies in the state like municipalities, district

boards, etc.,

• 1/12 are elected by graduates of three years standing and residing within the state,

• 1/12 are elected by teachers of three years standing in the state, not lower in

standard than secondary school,

• 1/3 are elected by the members of the legislative assembly of the state from amongst

persons who are not members of the assembly, and

• the remainder are nominated by the governor from amongst persons who have a

special knowledge or practical experience of literature, science, art, cooperative

movement and social service.

Thus, 5/6 of the total number of members of a legislative council are indirectly elected and

1/6 are nominated by the governor. The members are elected in accordance with the system

of proportional representation by means of a single transferable vote. The bonafide or

propriety of the governor’s nomination in any case cannot be challenged in the courts.

This scheme of composition of a legislative council as laid down in the Constitution is tentative

and not final. The Parliament is authorized to modify or replace the same. However, it has not

enacted any such law so far.

23. Cauvery Wildlife Sanctuary, sometime seen in the news, is located in which of the

following states?

(a) Kerala

(b) Tamil Nadu

(c) Karnataka

(d) None of the above

Page 22: SIMPLYFYING IAS EXAM PREPARATIONDAY – 41 (InstaTest-41) 1. Consider the following statements regarding Janaushadhi Sugam app 1. It has been launched by Ministry of Health and Family

INSTA 75 Days REVISION PLAN for Prelims 2020 - InstaTests

www.insightsonindia.com 20 Insights IAS

Solution: C

• Cauvery Wildlife Sanctuary is a protected area located in the Mandya,

Chamarajanagar and Ramanagar districts of Karnataka. The Cauvery River flows

through the middle of this sanctuary. The sanctuary adjoins Dharmapuri forest of

Tamil Nadu.

24. Consider the following statements regarding independence of a high court

1. The judges of a high court are provided with the security of tenure.

2. The salaries, allowances, privileges, leave and pension of the judges of a high court

are determined from time to time by the state legislature.

3. The salaries, allowances and pensions of the judges are charged on the

consolidated fund of the state.

4. The chief justice of a high court can appoint officers and servants of the high court

without any interference from the executive.

Which of the statements given above is/are correct?

(a) 1 and 2 only

(b) 2 and 3 only

Page 23: SIMPLYFYING IAS EXAM PREPARATIONDAY – 41 (InstaTest-41) 1. Consider the following statements regarding Janaushadhi Sugam app 1. It has been launched by Ministry of Health and Family

INSTA 75 Days REVISION PLAN for Prelims 2020 - InstaTests

www.insightsonindia.com 21 Insights IAS

(c) 1, 2, 3 and 4

(d) 1 and 4 only

Solution: D

• The independence of a high court is very essential for the effective discharge of the

duties assigned to it. It should be free from the encroachments, pressures and

interferences of the executive (council of ministers) and the legislature. It should be

allowed to do justice without fear or favour.

The Constitution has made the following provisions to safeguard and ensure the independent

and impartial functioning of a high court.

Mode of Appointment

• The judges of a high court are appointed by the president (which means the cabinet)

in consultation with the members of the judiciary itself (i.e., chief justice of India and

the chief justice of the high court).

Security of Tenure

• The judges of a high court are provided with the security of tenure.

Fixed Service Conditions

• The salaries, allowances, privileges, leave and pension of the judges of a high court are

determined from time to time by the Parliament.

Expenses Charged on Consolidated Fund

• The salaries and allowances of the judges, the salaries, allowances and pensions of the

staff as well as the administrative expenses of a high court are charged on the

consolidated fund of the state. Thus, they are non-votable by the state legislature

(though they can be discussed by it). It should be noted here that the pension of a

high court judge is charged on the Consolidated Fund of India and not the state.

Ban on Practice after Retirement

• The retired permanent judges of a high court are prohibited from pleading or acting

in any court or before any authority in India except the Supreme Court and the other

high courts.

Power to Punish for its Contempt

• A high court can punish any person for its contempt.

Page 24: SIMPLYFYING IAS EXAM PREPARATIONDAY – 41 (InstaTest-41) 1. Consider the following statements regarding Janaushadhi Sugam app 1. It has been launched by Ministry of Health and Family

INSTA 75 Days REVISION PLAN for Prelims 2020 - InstaTests

www.insightsonindia.com 22 Insights IAS

Freedom to Appoint its Staff

• The chief justice of a high court can appoint officers and servants of the high court

without any interference from the executive.

25. Consider the following events

1. Fourth General election in India

2. Formation of Haryana State

3. Mysore named as Karnataka State

4. Meghalaya and Tripura become full states

Which of the following is the correct chronological order of the above?

(a) 2-1-4-3

(b) 4-3-2-1

(c) 2-3-4-1

(d) 4-1-2-3

Solution: A

• Formation of Haryana State – 1966

• Fourth General election in India – 1967

• Meghalaya and Tripura become full states – 1972

• Mysore named as Karnataka State – 1973

DAY – 42 (InstaTest-42)

26. Consider the following statements regarding Prime Minister’s Economic Advisory

Council (PMEAC)

1. It is a statutory body.

2. It analyses all the works related to Budget, economic survey and it monitor the

funds allocation to various ongoing centrally sponsored schemes.

Which of the statements given above is/are correct?

(a) 1 only

(b) 2 only

(c) Both 1 and 2

Page 25: SIMPLYFYING IAS EXAM PREPARATIONDAY – 41 (InstaTest-41) 1. Consider the following statements regarding Janaushadhi Sugam app 1. It has been launched by Ministry of Health and Family

INSTA 75 Days REVISION PLAN for Prelims 2020 - InstaTests

www.insightsonindia.com 23 Insights IAS

(d) Neither 1 nor 2

Solution: D

• Economic Advisory Council to the Prime Minister (PMEAC) is a non-constitutional,

non-permanent and independent body constituted to give economic advice to the

Government of India, specifically the Prime Minister.

• It is constituted with the prime and sole aim to analyse all critical issues, economic or

otherwise, referred to it by the prime minister and advising him thereon.

• It is mandate to give advice to prime minister on economic matters such as inflation,

GDP changes, export-import changes, creating supporting environment for increased

trade and commerce.

• Submit periodic reports to PM related to macroeconomic developments and issues

which will have implications of the economic policy.

• It Analyse any topics, issues assigned by the PM and provide advice to them.

27. Consider the following statements regarding state election commission

1. The Governor may make provision with respect to all matters relating to elections

to the panchayats.

2. The removal of state election commissioner is similar to that of the removal of a

judge of the state high court.

Which of the statements given above is/are correct?

(a) 1 only

(b) 2 only

(c) Both 1 and 2

(d) Neither 1 nor 2

Solution: B

• The superintendence, direction and control of the preparation of electoral rolls and

the conduct of all elections to the panchayats shall be vested in the state election

commission. It consists of a state election commissioner to be appointed by the

governor. His conditions of service and tenure of office shall also be determined by

the governor. He shall not be removed from the office except in the manner and on

the grounds prescribed for the removal of a judge of the state high court. His

conditions of service shall not be varied to his disadvantage after his appointment.

Page 26: SIMPLYFYING IAS EXAM PREPARATIONDAY – 41 (InstaTest-41) 1. Consider the following statements regarding Janaushadhi Sugam app 1. It has been launched by Ministry of Health and Family

INSTA 75 Days REVISION PLAN for Prelims 2020 - InstaTests

www.insightsonindia.com 24 Insights IAS

• The state legislature may make provision with respect to all matters relating to

elections to the panchayats.

28. Consider the following statements regarding UPSC

1. The UPSC consists of a chairman and not more than ten members.

2. The Supreme Court has held that if the government fails to consult UPSC, the

aggrieved public servant has no remedy in a court.

3. The Supreme Court held that a selection by the UPSC, confers right to the post

upon the candidate.

Which of the statements given above is/are correct?

(a) 1 and 2 only

(b) 2 and 3 only

(c) 1 and 3 only

(d) 1, 2 and 3

Solution: A

Composition

• The UPSC consists of a chairman and other members appointed by the president of

India. The Constitution, without specifying the strength of the Commission has left

the matter to the discretion of the president, who determines its composition.

Usually, the Commission consists of nine to eleven members including the chairman.

Further, no qualifications are prescribed for the Commission’s membership except

that one-half of the members of the Commission should be such persons who have

held office for at least ten years either under the Government of India or under the

government of a state. The Constitution also authorises the president to determine

the conditions of service of the chairman and other members of the Commission.

• The Supreme Court has held that if the government fails to consult UPSC in the

matters (mentioned above), the aggrieved public servant has no remedy in a court.

In other words, the court held that any irregularity in consultation with the UPSC or

acting without consultation does not invalidate the decision of the government. Thus,

the provision is directory and not mandatory.

• Similarly, the court held that a selection by the UPSC does not confer any right to the

post upon the candidate. However, the government is to act fairly and without

arbitrariness or mala fides.

Page 27: SIMPLYFYING IAS EXAM PREPARATIONDAY – 41 (InstaTest-41) 1. Consider the following statements regarding Janaushadhi Sugam app 1. It has been launched by Ministry of Health and Family

INSTA 75 Days REVISION PLAN for Prelims 2020 - InstaTests

www.insightsonindia.com 25 Insights IAS

29. Uighurs, a religious community sometime seen in the news, is largely found in which of

the following province?

(a) Rakhine State of Myanmar

(b) Yemen

(c) Border region of Syria and Iraq

(d) Xinjiang province in China

Solution: D

• The Uighurs are Turkic Muslims who regard themselves as culturally and ethnically

close to Central Asian nations.

• Their language is similar to Turkish and about 11 million of them lives in China’s

western Xinjiang region.

https://www.thehindu.com/news/international/un-demands-unfettered-access-for-china-

uighur-region-visit/article30931384.ece

30. Consider the following statements regarding Finance Commission

1. It makes recommendations on distribution of the net proceeds of taxes to be

shared between the Centre and the states.

2. The commission submits its report to the Ministry of Finance.

3. The recommendations made by the Finance Commission are only of advisory

nature.

Which of the statements given above is/are correct?

(a) 1 only

(b) 2 and 3 only

(c) 1 and 3 only

(d) 1, 2 and 3

Solution: C

The Finance Commission is required to make recommendations to the president of India on

the following matters:

Page 28: SIMPLYFYING IAS EXAM PREPARATIONDAY – 41 (InstaTest-41) 1. Consider the following statements regarding Janaushadhi Sugam app 1. It has been launched by Ministry of Health and Family

INSTA 75 Days REVISION PLAN for Prelims 2020 - InstaTests

www.insightsonindia.com 26 Insights IAS

1. The distribution of the net proceeds of taxes to be shared between the Centre and

the states, and the allocation between the states of the respective shares of such

proceeds.

2. The principles that should govern the grants-in-aid to the states by the Centre (i.e.,

out of the consolidated fund of India).

3. The measures needed to augment the consolidated fund of a state to supplement

the resources of the panchayats and the municipalities in the state on the basis of the

recommendations made by the state finance commission.

4. Any other matter referred to it by the president in the interests of sound finance.

5. The commission submits its report to the president. He lays it before both the Houses

of Parliament along with an explanatory memorandum as to the action taken on its

recommendations.

It must be clarified here that the recommendations made by the Finance Commission are

only of advisory nature and hence, not binding on the government. It is up to the Union

government to implement its recommendations on granting money to the states.

31. Consider the following statements regarding Composition of the GST Council

1. Chairperson: The Union Finance Minister

2. Vice-Chairperson: Union Minister of State in-charge of Revenue or Finance

3. Chairperson of the Central Board of Excise and Customs (CBEC) as a permanent

invitee (non-voting).

4. Other members: The Minister in-charge of Finance or Taxation or any other

Minister nominated by each state government.

Which of the statements given above is/are correct?

(a) 1, 2 and 3 only

(b) 2, 3 and 4 only

(c) 1, 3 and 4 only

(d) 1, 2, 3 and 4

Solution: C

GST Council

Composition of the council

The Council is a joint forum of the center and the states and consists of the following

members:

(a) The Union Finance Minister as the Chairperson

Page 29: SIMPLYFYING IAS EXAM PREPARATIONDAY – 41 (InstaTest-41) 1. Consider the following statements regarding Janaushadhi Sugam app 1. It has been launched by Ministry of Health and Family

INSTA 75 Days REVISION PLAN for Prelims 2020 - InstaTests

www.insightsonindia.com 27 Insights IAS

(b) The Union Minister of State in-charge of Revenue or Finance

(c) The Minister in-charge of Finance or Taxation or any other Minister nominated

by each state government.

The members of the Council from the states have to choose one amongst themselves to be

the Vice-Chairperson of the Council. They can also decide his term.

The Union Cabinet also decided to include the Chairperson of the Central Board of Excise and

Customs (CBEC) as a permanent invitee (non-voting) to all proceedings of the Council.

32. Consider the following statements regarding India and World Heritage Sites?

1. Jaipur city is one of the first Indian cultural site listed in World Heritage site list.

2. India has one mixed site.

3. Properties into the list is finalized by the UN General assembly

Which of the statements given above is/are correct?

(a) 1 and 3 only

(b) 2 only

(c) 2 and 3 only

(d) 1, 2 and3

Solution: B

The United Nations Educational, Scientific and Cultural Organization (UNESCO) World

Heritage Sites are important places of cultural or natural heritage as described in the

UNESCO World Heritage Convention, established in 1972.

• There are 38 World Heritage Sites located in India. These include 30 cultural sites,

seven natural sites and one mixed-criteria site.

• Jaipur city was recently added to World Heritage site list.

• Properties into the list is finalized by the World Heritage Committee.

Extra Reading:

• The World Heritage Committee selects the sites to be listed as UNESCO World

Heritage Sites, including the World Heritage List and the List of World Heritage in

Danger, defines the use of the World Heritage Fund and allocates financial assistance

upon requests from States Parties.

• It comprises representatives from 21 state parties that are elected by the General

Assembly of States Parties for a four-year term.

Page 30: SIMPLYFYING IAS EXAM PREPARATIONDAY – 41 (InstaTest-41) 1. Consider the following statements regarding Janaushadhi Sugam app 1. It has been launched by Ministry of Health and Family

INSTA 75 Days REVISION PLAN for Prelims 2020 - InstaTests

www.insightsonindia.com 28 Insights IAS

• These parties vote on decisions and proposals related to the World Heritage

Convention and World Heritage List.

http://whc.unesco.org/en/statesparties/IN

33. Consider the following statements regarding National Commission STs

1. The Commission presents an annual report to the Ministry of Tribal affairs.

2. The Commission is vested with the power to regulate its own procedure.

3. The Commission, while investigating, has all the powers of a civil court trying a

suit.

Which of the statements given above is/are correct?

(a) 2 only

(b) 2 and 3 only

(c) 1 and 3 only

(d) 1, 2 and 3

Solution: B

The National Commission for SCs and STs came into being consequent upon passing of the

65th Constitutional Amendment Act of 1990. The Commission was established under Article

338 of the Constitution with the objective of monitoring all the safeguards provided for the

SCs and STs under the Constitution or other laws.

• Hence, in order to safeguard the interests of the STs more effectively, it was proposed

to set up a separate National Commission for STs by bifurcating the existing combined

National Commission for SCs and STs. This was done by passing the 89th

Constitutional Amendment Act of 2003. This Act further amended Article 338 and

inserted a new Article 338-A in the Constitution.

• The Commission presents an annual report to the President. It can also submit a

report as and when it thinks necessary.

• The President places all such reports before the Parliament, along with a

memorandum explaining the action taken on the recommendations made by the

Commission. The memorandum should also contain the reasons for the non-

acceptance of any of such recommendations.

• The Commission is vested with the power to regulate its own procedure.

• The Commission, while investigating any matter or inquiring into any complaint, has

all the powers of a civil court trying a suit.

• The Central government and the state governments are required to consult the

Commission on all major policy matters affecting the STs.

Page 31: SIMPLYFYING IAS EXAM PREPARATIONDAY – 41 (InstaTest-41) 1. Consider the following statements regarding Janaushadhi Sugam app 1. It has been launched by Ministry of Health and Family

INSTA 75 Days REVISION PLAN for Prelims 2020 - InstaTests

www.insightsonindia.com 29 Insights IAS

34. Consider the following statements regarding National Commission for Backward Classes

1. The Commission consists of a chairperson, a vice-chairperson and three other

members.

2. Their conditions of service and tenure of office are also determined by the

Parliament.

Which of the statements given above is/are correct?

(a) 1 only

(b) 2 only

(c) Both 1 and 2

(d) Neither 1 nor 2

Solution: A

In the Mandal case judgement (1992), the Supreme Court directed the central government

to constitute a permanent statutory body to examine the complaints of under inclusion,

overinclusion or non-inclusion of any class of citizens in the list of backward classes.

Accordingly, the National Commission for Backward Classes (NCBC) was set up in 1993.

• Later, the 102nd Amendment Act of 2018 conferred a constitutional status on the

Commission. For this purpose, the amendment inserted a new Article 338-B in the

constitution.

• Hence, the Commission ceased to be a statutory body and became a constitutional

body.

• The Commission consists of a chairperson, a vice-chairperson and three other

members. They are appointed by the President by warrant under his hand and seal.

Their conditions of service and tenure of office are also determined by the President.

35. Consider the following statements

1. Bosporus strait connects the Black Sea with the Sea of Marmara.

2. Strait of Hormuz separates Oman and Iran.

Which of the statements given above is/are correct?

(a) 1 only

(b) 2 only

(c) Both 1 and 2

(d) Neither 1 nor 2

Page 32: SIMPLYFYING IAS EXAM PREPARATIONDAY – 41 (InstaTest-41) 1. Consider the following statements regarding Janaushadhi Sugam app 1. It has been launched by Ministry of Health and Family

INSTA 75 Days REVISION PLAN for Prelims 2020 - InstaTests

www.insightsonindia.com 30 Insights IAS

Solution: C

• Bosporus strait is a narrow, natural strait and an internationally significant waterway

located in northwestern Turkey. It forms part of the continental boundary between

Europe and Asia, and divides Turkey by separating Anatolia from Thrace.

• The Bosporus connects the Black Sea with the Sea of Marmara, and, by extension via

the Dardanelles, the Aegean and Mediterranean seas.

• The Strait of Hormuz is a strait between the Persian Gulf and the Gulf of Oman. It

provides the only sea passage from the Persian Gulf to the open ocean and is one of

the world’s most strategically important choke points.

Page 33: SIMPLYFYING IAS EXAM PREPARATIONDAY – 41 (InstaTest-41) 1. Consider the following statements regarding Janaushadhi Sugam app 1. It has been launched by Ministry of Health and Family

INSTA 75 Days REVISION PLAN for Prelims 2020 - InstaTests

www.insightsonindia.com 31 Insights IAS

36. Consider the following statements regarding Special Officer for Linguistic Minorities

1. The post was added under Seventh Constitutional Amendment Act of 1956.

2. It was added based on the recommendation of States Reorganization

Commission.

3. Constitution does not specify the qualifications, tenure, salaries, service

conditions of the Special Officer for Linguistic Minorities.

Which of the statements given above is/are correct?

(a) 1 and 2 only

(b) 2 and 3 only

(c) 1 and 3 only

(d) 1, 2 and 3

Solution: D

Originally, the Constitution of India did not make any provision with respect to the Special

Officer for Linguistic Minorities. Later, the States Reorganization Commission (1953–55)

made a recommendation in this regard. Accordingly, the Seventh Constitutional Amendment

Act of 1956 inserted a new Article 350-B in Part XVII of the Constitution.

This article contains the following provisions:

• There should be a Special Officer for Linguistic Minorities. He is to be appointed by

the President of India.

• It would be the duty of the Special Officer to investigate all matters relating to the

safeguards provided for linguistic minorities under the Constitution. He would report

to the President upon those matters at such intervals as the President may direct. The

President should place all such reports before each House of Parliament and send to

the governments of the states concerned.

• It must be noted here that the Constitution does not specify the qualifications,

tenure, salaries and allowances, service conditions and procedure for removal of the

Special Officer for Linguistic Minorities.

• At the Central level, the Commissioner falls under the Ministry of Minority Affairs.

Hence, he submits the annual reports or other reports to the President through the

Union Minority Affairs Minister.

37. Consider the following statements regarding Comptroller and Auditor General of India

(CAG)

1. He is the guardian of the public purse and controls the entire financial system of

the country.

Page 34: SIMPLYFYING IAS EXAM PREPARATIONDAY – 41 (InstaTest-41) 1. Consider the following statements regarding Janaushadhi Sugam app 1. It has been launched by Ministry of Health and Family

INSTA 75 Days REVISION PLAN for Prelims 2020 - InstaTests

www.insightsonindia.com 32 Insights IAS

2. Minister of Finance represents the CAG in Parliament (both Houses).

3. He can be removed by the president on same grounds and in the same manner as

a judge of the Supreme Court.

Which of the statements given above is/are correct?

(a) 1 and 2 only

(b) 3 only

(c) 1 and 3 only

(d) 1, 2 and 3

Solution: C

The Constitution of India (Article 148) provides for an independent office of the Comptroller

and Auditor General of India (CAG). He is the head of the Indian Audit and Accounts

Department. He is the guardian of the public purse and controls the entire financial system

of the country at both the levels–the Centre and the state.

• The CAG is appointed by the president of India by a warrant under his hand and seal.

• He holds office for a period of six years or up to the age of 65 years, whichever is

earlier. He can resign any time from his office by addressing the resignation letter to

the president. He can also be removed by the president on same grounds and in the

same manner as a judge of the Supreme Court. In other words, he can be removed

by the president on the basis of a resolution passed to that effect by both the Houses

of Parliament with special majority, either on the ground of proved misbehavior or

incapacity.

• Further, no minister can represent the CAG in Parliament (both Houses) and no

minister can be called upon to take any responsibility for any actions done by him.

38. Consider the following statements regarding India VIX

1. It is a volatility index that serves as a measure of market expectation of volatility

in the near term.

2. The movement in the VIX index reflects the overall market volatility expectations

over the last 30 days.

Which of the statements given above is/are correct?

(a) 1 only

(b) 2 only

(c) Both 1 and 2

Page 35: SIMPLYFYING IAS EXAM PREPARATIONDAY – 41 (InstaTest-41) 1. Consider the following statements regarding Janaushadhi Sugam app 1. It has been launched by Ministry of Health and Family

INSTA 75 Days REVISION PLAN for Prelims 2020 - InstaTests

www.insightsonindia.com 33 Insights IAS

(d) Neither 1 nor 2

Solution: A

India VIX is a volatility index that serves as a measure of market expectation of volatility in

the near term.

• Volatility signifies the rate and magnitude of change in the stock price or index value.

The movement in the VIX index reflects the overall market volatility expectations over

the next 30 days.

• India VIX index is not the first of its kind in the world. The VIX index was first created

by the Chicago Board Options Exchange (CBOE) and introduced in 1993 based on the

prices of S&P 500 index.

39. Consider the following statements regarding Solicitor General of India

1. Unlike the post of Attorney-General for India, which is a Constitutional, the

Solicitor General is statutory in nature.

2. The Solicitor General of India is the government’s chief legal advisor, and its

primary lawyer in the Supreme Court of India.

Which of the statements given above is/are correct?

(a) 1 only

(b) 2 only

(c) Both 1 and 2

(d) Neither 1 nor 2

Solution: A

Solicitor General of India

• In addition to the AG, there are other law officers of the Government of India. They

are the solicitor general of India and additional solicitor general of India. They assist

the AG in the fulfilment of his official responsibilities. It should be noted here that only

the office of the AG is created by the Constitution. In other words, Article 76 does not

mention about the solicitor general and additional solicitor general.

The Attorney General of India is the government’s chief legal advisor, and its primary lawyer

in the Supreme Court of India. The Solicitor General of India is the secondary law officer of

Page 36: SIMPLYFYING IAS EXAM PREPARATIONDAY – 41 (InstaTest-41) 1. Consider the following statements regarding Janaushadhi Sugam app 1. It has been launched by Ministry of Health and Family

INSTA 75 Days REVISION PLAN for Prelims 2020 - InstaTests

www.insightsonindia.com 34 Insights IAS

the country, assists the Attorney-General, and is himself assisted by several Additional

Solicitors General of India. Like the Attorney-General for India, the Solicitor General and the

Additional Solicitors General advise the government and appear on behalf of the Union of

India in terms of the Law Officers (Terms and Conditions) Rules, 1972.

However, unlike the post of Attorney-General for India, which is a Constitutional post under

Article 76, the posts of the Solicitor General and the Additional Solicitors General are merely

statutory.

• Appointments Committee of the Cabinet appoints the Solicitor General.

40. The appointment committee for chairman of National Human Rights Commission

consists

1. Prime minister

2. Speaker of the Lok Sabha

3. Chairman of the Rajya Sabha

4. Leaders of the Opposition in both the Houses

5. Central home minister

Select the correct answer using the code given below:

(a) 1, 2 and 4 only

(b) 1, 2, 3 and 5 only

(c) 1, 2, 4 and 5 only

(d) 1, 2, 3, 4 and 5

Solution: C

The National Human Rights Commission is a statutory (and not a constitutional) body. It was

established in 1993 under a legislation enacted by the Parliament, namely, the Protection of

Human Rights Act, 1993.

• The commission is a multi-member body consisting of a chairperson and five

members.

• The chairperson and members are appointed by the president on the

recommendations of a six-member committee consisting of the prime minister as its

head, the Speaker of the Lok Sabha, the Deputy Chairman of the Rajya Sabha, leaders

of the Opposition in both the Houses of Parliament and the Central home minister.

• Further, a sitting judge of the Supreme Court or a sitting chief justice of a high court

can be appointed only after consultation with the chief justice of India.

Page 37: SIMPLYFYING IAS EXAM PREPARATIONDAY – 41 (InstaTest-41) 1. Consider the following statements regarding Janaushadhi Sugam app 1. It has been launched by Ministry of Health and Family

INSTA 75 Days REVISION PLAN for Prelims 2020 - InstaTests

www.insightsonindia.com 35 Insights IAS

41. ‘Operation Samudra Maitri’, sometime seen in the news, is carried out to assist the

survivors of earthquake from which country?

(a) Indonesia

(b) Sri Lanka

(c) Maldives

(d) Mauritius

Solution: A

Operation Samudra Maitri is the relief effort launched by India to assist the victims of the

2018 Sulawesi earthquake and tsunami in Indonesia.

On 28 September 2018, a shallow earthquake struck in the neck of the Minahasa Peninsula,

Indonesia, with its epicentre located in the mountainous Donggala Regency, Central

Sulawesi.

42. Consider the following statements regarding Central Information Commission

1. It was constituted under the provisions of the Information Commission Act, 2005.

2. The Commission consists of a Chief Information Commissioner and not more than

ten Information Commissioners.

Which of the statements given above is/are correct?

(a) 1 only

(b) 2 only

(c) Both 1 and 2

(d) Neither 1 nor 2

Solution: B

• The Central Information Commission was established by the Central Government in

2005. It was constituted through an Official Gazette Notification under the provisions

of the Right to Information Act (2005). Hence, it is not a constitutional body. The

Central Information Commission is a high-powered independent body which inter alia

looks into the complaints made to it and decide the appeals. It entertains complaints

and appeals pertaining to offices, financial institutions, public sector undertakings,

etc., under the Central Government and the Union Territories.

Page 38: SIMPLYFYING IAS EXAM PREPARATIONDAY – 41 (InstaTest-41) 1. Consider the following statements regarding Janaushadhi Sugam app 1. It has been launched by Ministry of Health and Family

INSTA 75 Days REVISION PLAN for Prelims 2020 - InstaTests

www.insightsonindia.com 36 Insights IAS

• The Commission consists of a Chief Information Commissioner and not more than ten

Information Commissioners.

• They should be persons of eminence in public life with wide knowledge and

experience in law, science and technology, social service, management, journalism,

mass media or administration and governance. They should not be a Member of

Parliament or Member of the Legislature of any State or Union Territory. They should

not hold any other office of profit or connected with any political party or carrying on

any business or pursuing any profession.

• The Chief Information Commissioner and an Information Commissioner shall hold

office for such term as prescribed by the Central Government or until they attain the

age of 65 years, whichever is earlier. They are not eligible for reappointment.

43. Consider the following statements

1. With the enactment of CVC Act, 2003, the superintendence of Delhi Special Police

Establishment vests with the Central Government.

2. The Director of CBI has been provided security of two-year tenure in office by the

CVC Act, 2003.

Which of the statements given above is/are correct?

(a) 1 only

(b) 2 only

(c) Both 1 and 2

(d) Neither 1 nor 2

Solution: C

• The CBI is headed by a Director. He is assisted by a special director or an additional

director. Additionally, it has a number of joint directors, deputy inspector generals,

superintendents of police and all other usual ranks of police personnel. In total, it has

about 5000 staff members, about 125 forensic scientists and about 250 law officers.

• The Director of CBI as Inspector-General of Police, Delhi Special Police Establishment,

is responsible for the administration of the organization. With the enactment of CVC

Act, 2003, the superintendence of Delhi Special Police Establishment vests with the

Central Government save investigations of offences under the Prevention of

Corruption Act, 1988, in which, the superintendence vests with the Central Vigilance

Commission. The Director of CBI has been provided security of two-year tenure in

office by the CVC Act, 2003.

Page 39: SIMPLYFYING IAS EXAM PREPARATIONDAY – 41 (InstaTest-41) 1. Consider the following statements regarding Janaushadhi Sugam app 1. It has been launched by Ministry of Health and Family

INSTA 75 Days REVISION PLAN for Prelims 2020 - InstaTests

www.insightsonindia.com 37 Insights IAS

44. Consider the following statements regarding International Fund for Agricultural

Development (IFAD)

1. It aims to double the farmer’s income through organic farming.

2. It is a member of the United Nations Development Group.

3. India is a founder member of IFAD.

Which of the statements given above is/are correct?

(a) 1 and 2 only

(b) 2 and 3 only

(c) 1 and 3 only

(d) 1, 2 and 3

Solution: B

International Fund for Agricultural Development (IFAD)

• The International Fund for Agricultural Development is an international financial

institution and a specialised agency of the United Nations that works to address

poverty and hunger in rural areas of developing countries.

• It is a specialized agency of the United Nations.

• It was established as an international financial institution in 1977 through United

Nations General Assembly Resolution as one of the major outcomes of the 1974

World Food Conference.

• Seventy-five percent of the world’s poor live in rural areas in developing countries, yet

only 4% of official development assistance goes to agriculture.

• The strategic policy of IFAD is detailed in Strategic Framework for IFAD 2011–2015:

Enabling the Rural Poor to Overcome Poverty.

• Its goal is to empower poor rural men and women in developing countries to achieve

higher incomes and improved food security.

• IFAD is managed by the two main governing bodies i.e. the Governing Council and the

Executive Board. The Governing Council is the highest decision-making body of IFAD.

• India is a founder member of IFAD and a key contributor among the member

countries.

45. Which of the following are the drawbacks of the Lokpal and Lokayuktas Act, 2013

1. Lokpal cannot suo motu proceed against any public servant.

2. Anonymous complaints not allowed.

3. Limitation period of 1 years to file complaints.

4. Heavy punishment for false and frivolous complaints.

Page 40: SIMPLYFYING IAS EXAM PREPARATIONDAY – 41 (InstaTest-41) 1. Consider the following statements regarding Janaushadhi Sugam app 1. It has been launched by Ministry of Health and Family

INSTA 75 Days REVISION PLAN for Prelims 2020 - InstaTests

www.insightsonindia.com 38 Insights IAS

Select the correct answer using the code given below:

(a) 1, 2 and 4 only

(b) 2, 3 and 4 only

(c) 1, 3 and 4 only

(d) 1, 2, 3 and 4

Solution: A

The following are the drawbacks (shortcomings) of the Lokpal and Lokayuktas Act, 2013:

• Lokpal cannot suo motu proceed against any public servant.

• Emphasis on form of complaint rather than substance.

• Heavy punishment for false and frivolous complaints against public servants may deter

complaints being filed to Lokpal.

• Anonymous complaints not allowed -Can’t just make a complaint on plain paper and

drop it in a box with supporting documents.

• Legal assistance to public servant against whom complaint is filed.

• Limitation period of 7 years to file complaints.

• Very non-transparent procedure for dealing with complaints against the PM.

46. Consider the following statements regarding Lunar Evacuation System Assembly (LESA)

1. It is a part of NASA’s 2024 Moon mission

2. It is a pyramid-like structure whose purpose is to rescue an astronaut should he

or she suffer an injury on the lunar surface.

3. It has been developed by NASA

Which of the statements given above is/are correct?

(a) 2 only

(b) 2 and 3 only

(c) 1 and 2 only

(d) 1, 2 and 3

Solution: C

Page 41: SIMPLYFYING IAS EXAM PREPARATIONDAY – 41 (InstaTest-41) 1. Consider the following statements regarding Janaushadhi Sugam app 1. It has been launched by Ministry of Health and Family

INSTA 75 Days REVISION PLAN for Prelims 2020 - InstaTests

www.insightsonindia.com 39 Insights IAS

Lunar Evacuation System Assembly (LESA):

• Among preparations for NASA’s 2024 Moon mission, one has been to test a device

called Lunar Evacuation System Assembly, or LESA.

• Developed by the European Space Agency (ESA)

• LESA is a pyramid-like structure whose purpose is to rescue an astronaut should he

or she suffer an injury on the lunar surface.

• LESA can be operated by a single astronaut to rescue a fallen colleague. It enables an

astronaut to lift their crewmate onto a mobile stretcher in less than 10 minutes,

before carrying them to the safety of a nearby pressurised lander.

47. Consider the following statements regarding National Disaster Management Authority

(NDMA)

1. The Minister of Home Affairs is the ex-officio chairperson of the NDMA.

2. It works under the administrative control of the Union Ministry of Home Affairs.

Which of the statements given above is/are correct?

(a) 1 only

(b) 2 only

(c) Both 1 and 2

(d) Neither 1 nor 2

Solution: B

Disaster Management Act, 2005.

• The Act provided for the creation of the National Disaster Management Authority

(NDMA) to spearhead and implement a holistic and integrated approach to disaster

management in the country. Initially, the NDMA was constituted in 2005 by an

Executive Order of the Government of India. Subsequently, the NDMA was notified in

2006 under the provisions of the Act.

• The NDMA consists of a chairperson and other members, not exceeding nine. The

Prime Minister is the ex-officio chairperson of the NDMA. The other members are

nominated by the chairperson of the NDMA. The chairperson of the NDMA designates

one of the members as the vice-chairperson of the NDMA. The vice chairperson has

the status of a Cabinet Minister while the other members have the status of a Minister

of State.

• The NDMA is the apex body for disaster management in the country. It works under

the administrative control of the Union Ministry of Home Affairs.

Page 42: SIMPLYFYING IAS EXAM PREPARATIONDAY – 41 (InstaTest-41) 1. Consider the following statements regarding Janaushadhi Sugam app 1. It has been launched by Ministry of Health and Family

INSTA 75 Days REVISION PLAN for Prelims 2020 - InstaTests

www.insightsonindia.com 40 Insights IAS

48. Consider the following statements regarding Langrangian point

1. It is a point where the attraction by the Sun and the Earth becomes equal.

2. Langrangian point experience the highest gravitational force.

Which of the statements given above is/are correct?

(a) 1 only

(b) 2 only

(c) Both 1 and 2

(d) Neither 1 nor 2

Solution: A

Lagrangian points are the locations in space where the combined gravitational pull of two

large masses roughly balance each other. Any small mass placed at that location will remain

at constant distances relative to the large masses. There are five such points in Sun-Earth

system and they are denoted as L1, L2, L3, L4 and L5. A halo orbit is a periodic three-

dimensional orbit near the L1, L2 or L3. It is a point where the attraction by the Sun and the

Earth becomes equal. The point doesn’t experience gravitational force.

Page 43: SIMPLYFYING IAS EXAM PREPARATIONDAY – 41 (InstaTest-41) 1. Consider the following statements regarding Janaushadhi Sugam app 1. It has been launched by Ministry of Health and Family

INSTA 75 Days REVISION PLAN for Prelims 2020 - InstaTests

www.insightsonindia.com 41 Insights IAS

49. Consider the following statements regarding All-India Services

1. The Central government create new all India services in consultation with the

state governments.

2. The all-India services are controlled jointly by the Central and state governments.

3. Any disciplinary action (imposition of penalties) against these officers can only be

taken by the Central government.

Which of the statements given above is/are correct?

(a) 2 only

(b) 2 and 3 only

(c) 3 only

(d) 1, 2 and 3

Solution: B

The All-India Services Act of 1951 authorised the Central government to make rules in

consultation with the state governments for the regulation of recruitment and service

conditions of the members of all-India services. The members of these services are recruited

and trained by the Central government but are assigned to different states for work. They

belong to different state cadres; the Centre having no cadre of its own in this regard.

The all-India services are controlled jointly by the Central and state governments. The

ultimate control lies with the Central government while the immediate control is vested in

the state governments. Any disciplinary action (imposition of penalties) against these officers

can only be taken by the Central government.

Article 312 makes the following provisions in respect of all-India services:

• The Parliament can create new all India services (including an all-India judicial service),

if the Rajya Sabha passes a resolution declaring that it is necessary or expedient in the

national interest to do so. Such a resolution in the Rajya Sabha should be supported

by two-thirds of the members present and voting. This power of recommendation is

given to the Rajya Sabha to protect the interests of states in the Indian federal system.

50. Consider the following statements regarding Financial Action Task Force (FATF)

1. It was established by a Group of Seven (G-7).

2. The FATF Secretariat is housed at the OECD headquarters in Paris.

3. The Asia/Pacific Group on Money Laundering works under FATF

Which of the pairs given above is/are correctly matched?

(a) 1 and 2 only

Page 44: SIMPLYFYING IAS EXAM PREPARATIONDAY – 41 (InstaTest-41) 1. Consider the following statements regarding Janaushadhi Sugam app 1. It has been launched by Ministry of Health and Family

INSTA 75 Days REVISION PLAN for Prelims 2020 - InstaTests

www.insightsonindia.com 42 Insights IAS

(b) 2 and 3 only

(c) 1 and 3 only

(d) 1, 2 and 3

Solution: A

Financial Action Task Force (FATF):

• The Financial Action Task Force (FATF) is an inter-governmental body established in

1989 on the initiative of the G7.

• It is a “policy-making body” which works to generate the necessary political will to

bring about national legislative and regulatory reforms in various areas.

• The FATF Secretariat is housed at the OECD headquarters in Paris.

• Objectives: The objectives of the FATF are to set standards and promote effective

implementation of legal, regulatory and operational measures for combating money

laundering, terrorist financing and other related threats to the integrity of the

international financial system.

• Functions: The FATF monitors the progress of its members in implementing necessary

measures, reviews money laundering and terrorist financing techniques and counter-

measures and promotes the adoption and implementation of appropriate measures

globally. In collaboration with other international stakeholders, the FATF works to

identify national-level vulnerabilities with the aim of protecting the international

financial system from misuse.

The Asia/Pacific Group on Money Laundering is an inter-governmental organisation,

consisting of 41 member jurisdictions, focused on ensuring that its members effectively

implement the international standards against money laundering, terrorist financing and

proliferation financing related to weapons of mass destruction. It doesn’t works under FATF.

DAY – 43 (InstaTest-43)

51. Know India Programme has been launched by

(a) NITI Aayog

(b) Ministry of Culture

(c) Ministry of Human Resource Development

(d) Ministry of External Affairs

Page 45: SIMPLYFYING IAS EXAM PREPARATIONDAY – 41 (InstaTest-41) 1. Consider the following statements regarding Janaushadhi Sugam app 1. It has been launched by Ministry of Health and Family

INSTA 75 Days REVISION PLAN for Prelims 2020 - InstaTests

www.insightsonindia.com 43 Insights IAS

Solution: D

• Know India Programme is a flagship programme of Ministry of External Affairs for

engagement with Indian origin youth (between 18-30 years) to enhance their

awareness about India, its cultural heritage, art and to familiarise them with various

aspects of contemporary India.

• Eligibility: Minimum qualification required for participating in KIP is graduation from

a recognized University /Institute or enrolled for graduation and ability to speak in

English. The applicant should not have visited India through any previous Programme

of Government of India. Those who have not visited India before will be given

preference.

52. Consider the following statements regarding Small Farmers Agribusiness Consortium

(SFAC)

1. It is an Autonomous Society registered under Societies Registration Act XXI of

1860.

2. It is governed by Board of Management which is chaired by Secretary, Department

of Agriculture, Cooperation and Farmers Welfare.

3. Society has been entrusted with the task of implementation of the National

Agriculture Market Scheme.

Which of the statements given above is/are correct?

(a) 1 only

(b) 2 and 3 only

(c) 1 and 3 only

(d) 1, 2 and 3

Solution: C

Small Farmers Agribusiness Consortium (SFAC) is an Autonomous Society promoted by

Ministry of Agriculture, Cooperation and Farmers’ Welfare, Government of India. It was

registered under Societies Registration Act XXI of 1860 on 18th January, 1994

• The Society is governed by Board of Management which is chaired, ex-officio, by

Hon’ble Union Minister for Agriculture and Farmers Welfare as the President and the

Secretary, Department of Agriculture, Cooperation and Farmers Welfare, Government

of India, is the ex-officio Vice-President.

Page 46: SIMPLYFYING IAS EXAM PREPARATIONDAY – 41 (InstaTest-41) 1. Consider the following statements regarding Janaushadhi Sugam app 1. It has been launched by Ministry of Health and Family

INSTA 75 Days REVISION PLAN for Prelims 2020 - InstaTests

www.insightsonindia.com 44 Insights IAS

• SFAC is implementing the central schemes of Government of India namely VCA, EGCGS

for economic inclusion of small and marginal farmers in agribusiness activities.

• Society is pioneer in organising small and marginal farmers as Farmers Interest

Groups, Farmers Producers Organisation and Farmers Producers Company for

endowing them with bargaining power and economies of scale. It provides a platform

for increased accessibility and cheaper availability of agricultural inputs to small and

marginal farmers and in establishing forward and backward linkages in supply chain

management. This initiative has triggered mobilization of farmers for aggregation

across the country with ultimate aim of sustainable business model and augmented

incomes.

• Recently the Society has been entrusted with the task of implementation of the

critically important Delhi KisanMandi and National Agriculture Market Scheme on e-

platform to progressively free agricultural trade and offer price discovery to farmers.

53. Consider the following statements regarding e-RaKAM

1. It is a first-of-its-kind initiative that leverages technology to connect farmers of

the smallest villages to the biggest markets of the world.

2. The portal is a joint initiative by state-run-auctioneer MSTC limited and Central

Warehousing Corporation.

Which of the statements given above is/are correct?

(a) 1 only

(b) 2 only

(c) Both 1 and 2

(d) Neither 1 nor 2

Solution: C

• Govt launches e-RaKAM portal for selling agri produce

Key facts:

• E-RaKAM is a first-of-its-kind initiative that leverages technology to connect farmers

of the smallest villages to the biggest markets of the world through internet and e-

RaKAM centres.

• The portal is a joint initiative by state-run-auctioneer MSTC and Central Warehousing

Corporation arm CRWC.

• E-RaKAM is developed by MSTC Limited and supported by marketing & logistics

partner CRWC Limited.

Page 47: SIMPLYFYING IAS EXAM PREPARATIONDAY – 41 (InstaTest-41) 1. Consider the following statements regarding Janaushadhi Sugam app 1. It has been launched by Ministry of Health and Family

INSTA 75 Days REVISION PLAN for Prelims 2020 - InstaTests

www.insightsonindia.com 45 Insights IAS

• E-RaKAM is a digital initiative bringing together the farmers, FPOs, PSUs, civil

supplies and buyers on a single platform to ease the selling and buying process of

agricultural products.

• Under this initiative, e-RaKAM centres are being developed in a phased manner

throughout the country to facilitate farmers for online sale of their produce.

54. Consider the following statements regarding Indian Council of World Affairs (ICWA)

1. It is a non-profit organisation established under Registration of Societies Act 1860

2. It is devoted exclusively for the study of international relations and foreign affairs.

3. Minister of External Affairs is the ex-officio President of ICWA.

Which of the statements given above is/are correct?

(a) 2 only

(b) 2 and 3 only

(c) 1 and 2 only

(d) 1, 2 and 3

Solution: C

• Dr. TCA Raghavan is the new Director General (DG) of Indian Council of World Affairs

(ICWA).

• Appointment: He was selected by Governing Body and Governing Council of ICWA

headed by Vice President M. Venkaiah Naidu.

Indian Council of World Affairs (ICWA):

• It was established in 1943 by group of Indian intellectuals as think tank. It was

established as non-official, non-political and non-profit organisation under

Registration of Societies Act 1860.

• It was declared institution of national importance by Indian Council of World Affairs

(ICWA), 2001 enacted by Parliament.

• The Vice President of India is the ex-officio President of ICWA, while the Minister of

External Affairs is its Vice-President.

• ICWA is devoted exclusively for the study of international relations and foreign

affairs.

• It had conducted historic international conferences like Asian Relations Conference

in 1947 under leadership Sarojini Naidu and United Nations and New World Order in

1994.

Page 48: SIMPLYFYING IAS EXAM PREPARATIONDAY – 41 (InstaTest-41) 1. Consider the following statements regarding Janaushadhi Sugam app 1. It has been launched by Ministry of Health and Family

INSTA 75 Days REVISION PLAN for Prelims 2020 - InstaTests

www.insightsonindia.com 46 Insights IAS

55. Consider the following statements regarding Pharma Jan Samadhan (PJS) initiative

1. It aims at empowering the citizen through a transparent, accountable and

responsive governance system.

2. It would serve as a robust e-governance tool for protection of consumers’

interests through effective implementation of the Drugs (Price Control) Order

2013.

3. It was created by National Informatics Centre.

Which of the statements given above is/are correct?

(a) 1 and 2 only

(b) 2 and 3 only

(c) 1 only

(d) 1, 2 and 3

Solution: A

The Pharma Jan Samadhan (PJS) initiative is part of Government of India’s Digital India

initiative, which aims at empowering the citizen through a transparent, accountable and

responsive governance system. The PJS provides the consumer with an effective and time

bound online grievance redressal system to effectively deal with complaints related to pricing,

shortage and non-availability of medicines.

Pharma Jan Samadhan scheme

• Pharma Jan Samadhan Scheme was recently launched by the Union Minister of

Chemicals & Fertilizers.

What is it?

• It is a web enabled system for redressal of consumers’ grievances relating to pricing

and availability of medicines. It was created by National Pharmaceutical Pricing

Authority (NPPA)

Details:

• The ‘Pharma Jan Samadhan’ scheme has put in place a speedy and effective complaint

redressal system with respect to availability and pricing of medicines.

• It would serve as a robust e-governance tool for protection of consumers’ interests

through effective implementation of the Drugs (Price Control) Order 2013.

• It will provide consumers and others with an on-line facility to redress their complaints

relating to over-pricing of medicines, non-availability or shortage of medicines, sale of

new medicines without prior price approval of NPPA, and refusal of supply for sale of

Page 49: SIMPLYFYING IAS EXAM PREPARATIONDAY – 41 (InstaTest-41) 1. Consider the following statements regarding Janaushadhi Sugam app 1. It has been launched by Ministry of Health and Family

INSTA 75 Days REVISION PLAN for Prelims 2020 - InstaTests

www.insightsonindia.com 47 Insights IAS

any medicine without good and sufficient reason. NPPA will initiate action on any

complaint within 48 hrs of its receipt.

56. Consider the following statements regarding e-Biz

1. e-Biz is a part of the 27 Mission Mode Projects (MMPs) under the National E-

Governance Plan (NEGP).

2. e-Biz is being implemented by Wipro Technologies Limited.

3. It aims to improve the business environment in the country by enabling fast and

efficient access to Government-to-Business (G2B) services.

Which of the statements given above is/are correct?

(a) 1 and 2 only

(b) 3 only

(c) 1 and 3 only

(d) 1, 2 and 3

Solution: C

About e-Biz:

• e-Biz is one of the integrated services projects and part of the 27 Mission Mode

Projects (MMPs) under the National E-Governance Plan (NEGP) of the Government

of India.

• Implementation: e-Biz is being implemented by Infosys Technologies Limited (Infosys)

under the guidance and aegis of Department of Industrial Policy and Promotion

(DIPP), Ministry of Commerce & Industry, Government of India.

• The focus of e-Biz is to improve the business environment in the country by enabling

fast and efficient access to Government-to-Business (G2B) services through an online

portal. This will help in reducing unnecessary delays in various regulatory processes

required to start and run businesses.

• Aim: This project aims at creating an investor-friendly business environment in India

by making all regulatory information – starting from the establishment of a business,

through its ongoing operations, and even its possible closure – easily available to the

various stakeholders concerned. In effect, it aims to develop a transparent, efficient

and convenient interface, through which the government and businesses can interact

in a timely and cost-effective manner, in the future.

Page 50: SIMPLYFYING IAS EXAM PREPARATIONDAY – 41 (InstaTest-41) 1. Consider the following statements regarding Janaushadhi Sugam app 1. It has been launched by Ministry of Health and Family

INSTA 75 Days REVISION PLAN for Prelims 2020 - InstaTests

www.insightsonindia.com 48 Insights IAS

57. Consider the following statements regarding Mid-Day Meal Scheme

1. The Midday Meal Scheme is covered by the National Food Security Act, 2013.

2. It is implemented by the Ministry of Health and Family Welfare.

3. The scheme covers all school children studying in Classes I-VIII of Government,

Government-Aided Schools.

Which of the statements given above is/are correct?

(a) 1 only

(b) 2 and 3 only

(c) 1 and 3 only

(d) 1, 2 and 3

Solution: C

Mid-Day Meal Scheme (MDM)

• It was started in 1995 as National Programme of Nutritional Support to Primary

Education.

• The Mid-Day Meal Scheme is implemented by the Ministry of Human Resource

Development.

• The Midday Meal Scheme is a school meal programme of the Government of India

designed to improve the nutritional status of school-age children nationwide

• Under the Convention on the Rights of the Child, to which India is a party, India has

committed to provide “adequate nutritious foods” for children.

• The main objective of the Mid-Day Meal Scheme is to attain the goal of

universalization of primary education.

• The Midday Meal Scheme is covered by the National Food Security Act, 2013.

• Mid-Day Meal Scheme (MDMS), is a Centrally-Sponsored Scheme which covers all

school children studying in Classes I-VIII of Government, Government-Aided Schools.

58. Consider the following statements regarding Mission Raksha Gyan Shakti

1. It was initiated by Ministry of Commerce and Industry.

2. It aims to provide a boost to the IPR culture in indigenous defense industry.

3. The Directorate General of Quality Assurance (DGQA) has been entrusted with

the responsibility of coordinating and implementing the programme.

Which of the statements given above is/are correct?

(a) 1 and 2 only

(b) 2 and 3 only

Page 51: SIMPLYFYING IAS EXAM PREPARATIONDAY – 41 (InstaTest-41) 1. Consider the following statements regarding Janaushadhi Sugam app 1. It has been launched by Ministry of Health and Family

INSTA 75 Days REVISION PLAN for Prelims 2020 - InstaTests

www.insightsonindia.com 49 Insights IAS

(c) 1 and 3 only

(d) 1, 2 and 3

Solution: B

Raksha Mantri Launches ‘Mission Raksha Gyan Shakti’

• Raksha Mantri Smt Nirmala Sitharaman has formally launched ‘Mission Raksha Gyan

Shakti’.

• The event showcased salient inventions and innovations achieved by Defence

Research and Development Organisation (DRDO), Defence Public Sector

Undertakings (DPSUs), and Ordinance Factories (OFs) which have resulted in

successful filing of Intellectual Property Rights (IPR) applications.

• The Directorate General of Quality Assurance (DGQA) has been entrusted with the

responsibility of coordinating and implementing the programme.

Objective:

• As part of the ongoing initiatives to enhance self-reliance in defence, the Mission

Raksha Gyan Shakti aims to provide a boost to the IPR culture in indigenous defence

industry.

Other Highlights:

• IPR emerged as a key ingredient of an ecosystem which stimulates innovation and

ingenuity.

• It was highlighted the need to migrate from the culture of seeking Transfer of

Technology (ToT) from foreign sources to generating Intellectual Property in India, to

achieve the goal of self-reliance in Defence sector.

IP Facilitation Cell:

It was established in order

• To achieve ambitious targets of training 10,000 personnel of Ordnance Factories (OFs)

and Defence Public Sector Undertakings (DPSUs) on IPR.

• To facilitate filing of at least 1,000 new IPR applications.

59. Consider the following statements regarding Atmosphere & Climate Research-

Modelling Observing Systems & Services (ACROSS) scheme

1. It will be implemented by the Ministry of Earth Sciences.

2. The scheme will aim at improving skill of weather and climate forecast through

sustained observations, intensive R & D.

Page 52: SIMPLYFYING IAS EXAM PREPARATIONDAY – 41 (InstaTest-41) 1. Consider the following statements regarding Janaushadhi Sugam app 1. It has been launched by Ministry of Health and Family

INSTA 75 Days REVISION PLAN for Prelims 2020 - InstaTests

www.insightsonindia.com 50 Insights IAS

Which of the statements given above is/are correct?

(a) 1 only

(b) 2 only

(c) Both 1 and 2

(d) Neither 1 nor 2

Solution: C

“Atmosphere & Climate Research-Modelling Observing Systems & Services (ACROSS)”

scheme

• The Cabinet Committee has approved continuation of the nine sub-schemes of the

umbrella scheme “Atmosphere & Climate Research-Modelling Observing Systems &

Services (ACROSS)” during 2017-2020.

• It will be implemented by the Ministry of Earth Sciences through its institutes namely

India Meteorological Department (IMD), Indian Institute of Tropical Meteorology

(IITM), National Centre for Medium Range Weather Forecasting (NCMRWF), and

Indian National Centre for Ocean Information Service (INCOIS).

Background:

• The Ministry of Earth Sciences has a mandate to carry out research and development

activities to develop and improve capability to forecast weather, climate and natural

hazard related phenomena. Towards this direction, MoES has taken several initiatives

to formulate specific schemes like weather and climate modelling, monsoon-research,

climate change science & climate services etc.

• These schemes involve multi-institutes wherein each unit has a designated role for

accomplishing the aforesaid tasks. As a result, all these schemes with specific

objectives and budget are implemented in an integrated manner and are put together

under the umbrella scheme “ACROSS”.

ACROSS Scheme:

• ACROSS scheme pertains to the atmospheric science programs of the Ministry of

Earth Sciences (MoES).

• It addresses different aspects of weather and climate services, which includes

warnings for cyclone, storm surges, heat waves, thunderstorms etc.

• Each of these aspects is incorporated as nine sub-schemes under the umbrella scheme

“ACROSS” and is implemented in an integrated.

• The ACROSS scheme consists of nine sub-programmes which are multi-disciplinary

and multi institutional in nature and will be implemented in an integrated manner.

Page 53: SIMPLYFYING IAS EXAM PREPARATIONDAY – 41 (InstaTest-41) 1. Consider the following statements regarding Janaushadhi Sugam app 1. It has been launched by Ministry of Health and Family

INSTA 75 Days REVISION PLAN for Prelims 2020 - InstaTests

www.insightsonindia.com 51 Insights IAS

• As the objective of the ACROSS scheme is to provide a reliable weather and climate

forecast for betterment of society, the scheme will aim at improving skill of weather

and climate forecast through sustained observations, intensive R & D.

Benefits of the Scheme:

• The scheme will provide improved weather, climate and ocean forecast and

services, thereby ensuring transfer of commensurate benefits to the various services.

• It will also provide a sizable number of scientific and technical staff along with requisite

administrative support, thereby generating employment.

• To ensure last-mile connectivity of the weather based services to the end -user, a large

number of agencies like the Krishi Vigyana Kendras of ICAR, Universities and local

municipalities are roped in thus generating employment opportunities to many

people.

60. Consider the following statements regarding Manas National Park

1. It is listed as a World Heritage Site by UNESCO.

2. The Manas River serves as an international border dividing India and Nepal.

3. It is famous for its rare and endangered endemic wildlife like Assam Roofed Turtle,

Hispid Hare, Golden Langur and Pygmy Hog.

Which of the statements given above is/are correct?

(a) 2 only

(b) 1 and 3 only

(c) 1 and 2 only

(d) 1, 2 and 3

Solution: B

Manas National Park (MNP):

• Manas National Park is a national park, UNESCO Natural World Heritage site, a Project

Tiger reserve, an elephant reserve and a biosphere reserve in Assam, India.

• Located in the Himalayan foothills, it is contiguous with the Royal Manas National

Park in Bhutan.

• The Manas river flows thorough the west of the park and is the main river within it. It

is a major tributary of Brahmaputra river and splits into two separate rivers, the Beki

and Bholkaduba as it reaches the plains.

• The Manas River also serves as an international border dividing India and Bhutan.

Page 54: SIMPLYFYING IAS EXAM PREPARATIONDAY – 41 (InstaTest-41) 1. Consider the following statements regarding Janaushadhi Sugam app 1. It has been launched by Ministry of Health and Family

INSTA 75 Days REVISION PLAN for Prelims 2020 - InstaTests

www.insightsonindia.com 52 Insights IAS

• The bedrock of the savanna area in the north of the park is made up of limestone and

sandstone, whereas the grasslands in the south of the park stand on deep deposits of

fine alluvium.

• The combination of Sub-Himalayan Bhabar Terai formation along with the riverine

succession continuing up to Sub-Himalayan mountain forest make it one of the richest

areas of biodiversity in the world.

• The fauna of the sanctuary include Indian elephants, Indian rhinoceros, gaurs, Asian

water buffaloes, barasingha, Indian tigers, Indian leopards, clouded leopards, Asian

golden cats, dholes, capped langurs, golden langurs, Assamese macaques, slow loris,

hoolock gibbons, smooth-coated otters, sloth bears, barking deers, hog deers, black

panthers, sambar deers and chitals.

• The park is well known for species of rare and endangered wildlife that are not found

anywhere else in the world like the Assam roofed turtle, hispid hare, golden langur

and pygmy hog.

61. Consider the following statements regarding Saksham initiative

1. It is an initiative of Ministry of New and Renewable Energy.

2. It an annual one-month long, people-centric fuel conservation mega campaign of

Petroleum Conservation Research Association (PCRA).

Which of the statements given above is/are correct?

(a) 1 only

(b) 2 only

(c) Both 1 and 2

(d) Neither 1 nor 2

Solution: B

Saksham

• It is a people centric fuel conservation mega campaign of Petroleum Conservation

Research Association (PCRA).

Saksham actively involves the Oil & Gas PSUs along with other stakeholders like State

Governments:

• To create focused attention on fuel conservation through people centric activities and

• To sensitize the masses about the conservation and efficient use of petroleum

products leading to better health and environment.

Page 55: SIMPLYFYING IAS EXAM PREPARATIONDAY – 41 (InstaTest-41) 1. Consider the following statements regarding Janaushadhi Sugam app 1. It has been launched by Ministry of Health and Family

INSTA 75 Days REVISION PLAN for Prelims 2020 - InstaTests

www.insightsonindia.com 53 Insights IAS

About PCRA (established in 1978):

• It is a registered society set up under the aegis of Ministry of Petroleum & Natural

Gas, Government of India.

• As a non-profit organization, PCRA is a national government agency engaged in

promoting energy efficiency in various sectors of economy.

• ‘Saksham’, an annual one-month long, people-centric fuel conservation mega

campaign of Petroleum Conservation Research Association (PCRA) under the aegis of

Ministry of Petroleum and Natural Gas is going to be launched on 16th January by

Minister of Petroleum & Natural Gas and Steel Shri Dharmendra Pradhan in a function

to be held in Delhi.

62. Consider the following statements regarding the Gandhian Challenge

1. It was launched by Ministry of Culture.

2. This innovation challenge provides a platform for every child across India to ideate

innovative solutions for a sustainable India of their dreams.

Which of the statements given above is/are correct?

(a) 1 only

(b) 2 only

(c) Both 1 and 2

(d) Neither 1 nor 2

Solution: B

The Gandhian Challenge

• On the 150th birth Anniversary of Mahatma Gandhi, AIM, NITI Aayog’s Atal

Tinkering Labs (ATL) and UNICEF India, including Generation Unlimited, have

launched ‘The Gandhian Challenge’.

What is it?

• This innovation challenge provides a platform for every child across India to ideate

innovative solutions for a sustainable India of their dreams, using Gandhi’s principles.

• The winners of The Gandhian Challenge will be awarded in New Delhi by NITI Aayog’s

Atal Innovation Mission and UNICEF on the occasion of Children’s Day in November.

• The contest – open for every child in India from 2 October to 20 October – also

celebrates 70 years of partnership between Government of India and UNICEF India to

enable Every Right for Every Child.

Page 56: SIMPLYFYING IAS EXAM PREPARATIONDAY – 41 (InstaTest-41) 1. Consider the following statements regarding Janaushadhi Sugam app 1. It has been launched by Ministry of Health and Family

INSTA 75 Days REVISION PLAN for Prelims 2020 - InstaTests

www.insightsonindia.com 54 Insights IAS

• Ideas and solutions to the Gandhian Challenge may be expressed through broad

categories: Art & Innovation (Letters, poems, painting, videos and photos, among

others) and Science, Technology & Innovation (Robotics, IoT, sensors and 3D printers,

among others).

63. Under which of the following acts the Sentinelese tribes have been protected?

1. A &N Islands (PAT) Regulation 1956.

2. Restrictions under Foreigner (Restricted Area) Orders, 1963.

3. Indian Forest Act, 1927

4. Wildlife (Protection) Act, 1972.

5. Scheduled Castes and the Scheduled Tribes (Prevention of Atrocities) Act, 1989.

Select the correct answer from the codes given below

(a) 1, 2, and 3 only

(b) 1, 2, 4 and 5 only

(c) 1, 3, and 5 only

(d) 1, 2, 3, 4, and 5

Solution: D

Sentinelese:

• The Sentinelese is a negrito tribe who live on the North Sentinel Island of the

Andamans. The inhabitants are connected to the Jarawa on the basis of physical, as

well as linguistic similarities. Their numbers are believed to be less than 150 and as

low as 40.

• Based on carbon dating of kitchen middens by the Anthropological Survey of India,

Sentinelese presence was confirmed in the islands to 2,000 years ago. Genome studies

indicate that the Andaman tribes could have been on the islands even 30,000 years

ago.

• Surveys of North Sentinel Island have not found any evidence of agriculture. Instead,

the community seems to be hunter-gatherers, getting food through fishing, hunting,

and collecting wild plants living on the island.

• Sentinelese are also listed under Particularly Vulnerable Tribal Groups (PVTGs) by

the government of India.

They have been protected under:

• A &N Islands (PAT) Regulation 1956.

• Scheduled Castes and the Scheduled Tribes (Prevention of Atrocities) Act, 1989.

Page 57: SIMPLYFYING IAS EXAM PREPARATIONDAY – 41 (InstaTest-41) 1. Consider the following statements regarding Janaushadhi Sugam app 1. It has been launched by Ministry of Health and Family

INSTA 75 Days REVISION PLAN for Prelims 2020 - InstaTests

www.insightsonindia.com 55 Insights IAS

• Restrictions under Foreigner (Restricted Area) Orders, 1963.

• Visa Manual Conditions/Passport Act 1920,

• Indian Forest Act, 1927

• Wildlife (Protection) Act, 1972.

Steps taken to ensure the protection of Sentinelese:

• The entire North Sentinel Island along with 5 km coastal sea from high water mark is

notified as tribal reserve.

• The Government respects their way of life style, therefore, has adopted an ‘eyes-on

and hands-off’ practice to protect and safeguard the Sentinelese tribe.

• A protocol of circumnavigation of the North Sentinel Island has been notified. The

ships and aircrafts of Coast Guard and boats of Marine Police make sorties around

North Sentinel to keep surveillance.

64. Consider the following statements regarding Participatory Guarantee Scheme (PGS)

1. It is an initiative under Ministry of Agriculture

2. It is a process of certifying organic products, which ensures that their production

takes place in accordance with laid-down quality standards.

3. It operates outside the frame of third-party certification

Which of the statements given above is/are correct?

(a) 1 and 2 only

(b) 2 and 3 only

(c) 1 and 3 only

(d) 1, 2 and 3

Solution: D

Participatory Guarantee Scheme (PGS)

• Union Agriculture Ministry’s PGS is a process of certifying organic products, which

ensures that their production takes place in accordance with laid-down quality

standards.

• The certification is in the form of a documented logo or a statement.

• According to the International Federation of Organic Agriculture Movements

(IFOAM), the Bonn-based global umbrella organisation for the organic agriculture

movement, PGSs are “locally focused quality assurance systems” that “certify

producers based on active participation of stakeholders and are built on a foundation

of trust, social networks and knowledge exchange”.

Page 58: SIMPLYFYING IAS EXAM PREPARATIONDAY – 41 (InstaTest-41) 1. Consider the following statements regarding Janaushadhi Sugam app 1. It has been launched by Ministry of Health and Family

INSTA 75 Days REVISION PLAN for Prelims 2020 - InstaTests

www.insightsonindia.com 56 Insights IAS

PGS – India

• PGS-India (Participatory Guarantee System of India) is a quality assurance initiative

that is locally relevant, emphasize the participation of stakeholders, including

producers and consumers and operate outside the frame of third-party certification.

Guiding Principles

• PGS India system is based on participatory approach, a shared vision, transparency

and trust. In addition it gives PGS movement a National recognition and institutional

structure.

65. Consider the following statements regarding Janani Shishu Suraksha Karyakaram (JSSK)

1. Its objective is to eliminate out of pocket expenses for both pregnant women and

sick infants.

2. The initiative entitles all pregnant women delivering in public health institutions

absolutely free.

3. The initiative allows all sick newborns access to the public health institutions for

treatment till 90 days after birth.

Which of the statements given above is/are correct?

(a) 1 and 2 only

(b) 2 and 3 only

(c) 1 only

(d) 1, 2 and 3

Solution: A

Janani Shishu Suraksha Karyakaram (JSSK) has been launched with the objective to eliminate

out of pocket expenses for both pregnant women and sick infants accessing public health

institution for treatment.

• The initiative was estimated to benefit more than 1 crore pregnant women access

public health institutions every year in both urban and rural areas.

• In view of the difficulty being faced by the pregnant women and parents of sick new-

born along-with high out of pocket expenses incurred by them on delivery and

treatment of sick- new-born, Ministry of Health and Family Welfare (MoHFW) has

taken a major initiative to evolve a consensus on the part of all States to provide

completely free and cashless services to pregnant women including normal deliveries

and caesarean operations and sick new born (up to 30 days after birth) in Government

health institutions in both rural and urban areas.

Page 59: SIMPLYFYING IAS EXAM PREPARATIONDAY – 41 (InstaTest-41) 1. Consider the following statements regarding Janaushadhi Sugam app 1. It has been launched by Ministry of Health and Family

INSTA 75 Days REVISION PLAN for Prelims 2020 - InstaTests

www.insightsonindia.com 57 Insights IAS

Key features of the scheme

• The initiative entitles all pregnant women delivering in public health institutions to

absolutely free and no expense delivery, including caesarean section.

The entitlements include free drugs and consumables, free diet up to 3 days during normal

delivery and up to 7 days for C-section, free diagnostics, and free blood wherever required.

This initiative also provides for free transport from home to institution, between facilities in

case of a referral and drop back home. Similar entitlements have been put in place for all sick

newborns accessing public health institutions for treatment till 30 days after birth. This has

now been expanded to cover sick infants:

• The scheme aims to eliminate out of pocket expenses incurred by the pregnant

women and sick new borne while accessing services at Government health facilities.

• The scheme is estimated to benefit more than 12 million pregnant women who access

Government health facilities for their delivery. Moreover it will motivate those who

still choose to deliver at their homes to opt for institutional deliveries.

• All the States and UTs have initiated implementation of the scheme

66. Consider the following statements regarding Payment Banks

1. It has to maintain Cash Reserve Ratio (CRR)

2. M L Dantwala Committee had recommended to create Payment Banks

3. It can lend to a few selected category of customers such as migrant labour

workforce, low income households and small businesses.

Which of the statements given above is/are correct?

(a) 1 only

(b) 2 and 3 only

(c) 1 and 2 only

(d) 1, 2 and 3

Solution: A

Payment Banks

• Payments banks are the financial institutions with a differentiated license from the

RBI to provide selected financial services like saving deposits and current deposits and

payment facilities.

Objectives of payments banks are to promote financial inclusion by providing:

• small savings accounts and

Page 60: SIMPLYFYING IAS EXAM PREPARATIONDAY – 41 (InstaTest-41) 1. Consider the following statements regarding Janaushadhi Sugam app 1. It has been launched by Ministry of Health and Family

INSTA 75 Days REVISION PLAN for Prelims 2020 - InstaTests

www.insightsonindia.com 58 Insights IAS

• Payments / remittance services to the underserved population.

• Nachiket Mor Committee recommended creation of Payment Banks.

• It cannot offer all the services that a commercial bank offers.

• It can take deposits upto 1 lakh per account and it can issue debit cards but not credit

cards.

• It cannot lend.

• It has to maintain Cash Reserve Ratio (CRR).

• It can hold a maximum of 25% in current and time/fixed deposits with other

commercial banks for operational purposes and liquidity management.

• A payments bank can work as a business correspondent (BC) of another bank.

67. Consider the following statements

1. Under Ishan Vikas, scholarship is provided to the economically backward students

from the North East Region.

2. Under Ishan Uday, selected school children from North Eastern States are brought

in close contact with the Indian Institute of Technology (IITs), to motivate them to

pursue in STEM fields.

Which of the statements given above is/are correct?

(a) 1 only

(b) 2 only

(c) Both 1 and 2

(d) Neither 1 nor 2

Solution: D

• Ishan Vikas and Ishan Uday schemes are being implemented for the students of North

Eastern region.

• Ishan Vikas is coordinated by IIT, Guwahati and under the programme, selected school

children from North Eastern States are brought in close contact with the Indian

Institute of Technology (IITs), Indian Institutes of Science Education and Research

(IISERs) and National Institute of Advanced Studies (NIAS) during the vacation period

to motivate them to pursue science, technology, engineering and mathematics(STEM)

and to facilitate internship opportunities for the engineering college students in

various institutes of national importance. Annual target is to provide visit of 96 school

children to premier institutes and summer internships to 250 engineering students.

Rs. 605.45 lakhs has been approved for the programme. Since inception of the

programme in 2014-15, 1637 school children and 372 Engineering students have

benefited.

Page 61: SIMPLYFYING IAS EXAM PREPARATIONDAY – 41 (InstaTest-41) 1. Consider the following statements regarding Janaushadhi Sugam app 1. It has been launched by Ministry of Health and Family

INSTA 75 Days REVISION PLAN for Prelims 2020 - InstaTests

www.insightsonindia.com 59 Insights IAS

• Ishan Uday Scholarship Scheme is administered by University Grants Commission

(UGC). Under the scheme, scholarship is provided to the economically backward

students from the North East Region for pursuing general degree courses, technical

and professional degree courses. It is envisaged to provide 10000 scholarships

annually. Since inception of the scheme in 2014-15, Rs. 224.82 Crore has been

disbursed towards release of 47525 fresh and renewal scholarships.

68. Consider the following statements regarding Scheme of Fund for Regeneration of

Traditional Industries (SFURTI) Scheme

1. Ministry of Textiles is implementing the Scheme.

2. Under this, financial support is being provided for setting up of traditional

industries clusters.

3. Coir Board and Khadi and Village Industries Commission are the implementing

agency.

Which of the statements given above is/are correct?

(a) 1 and 2 only

(b) 2 and 3 only

(c) 1 and 3 only

(d) 1, 2 and 3

Solution: B

• Ministry of MSME is implementing a ‘Scheme of Fund for Regeneration of Traditional

Industries’ (SFURTI) Scheme under which financial support is being provided for

setting up of traditional industries clusters viz. Khadi, Coir & Village industries clusters.

The objectives of the SFURTI Scheme are:

• To develop clusters of traditional industries in the country over a period of five years.

• To make traditional industries more competitive, market-driven, productive and

profitable.

• To strengthen the local governance system of industry clusters, with active

participation of the local stakeholders, so that they are enabled to development

initiatives.

• To build up innovated and traditional skills, improved technologies, advanced

processes, market intelligence and new models of public-private partnerships, so as

to gradually replicate similar models of cluster-based regenerated traditional

industries.

Page 62: SIMPLYFYING IAS EXAM PREPARATIONDAY – 41 (InstaTest-41) 1. Consider the following statements regarding Janaushadhi Sugam app 1. It has been launched by Ministry of Health and Family

INSTA 75 Days REVISION PLAN for Prelims 2020 - InstaTests

www.insightsonindia.com 60 Insights IAS

The following agencies shall be designated as the Nodal Agencies for the Scheme:

• Khadi and Village Industries Commission – for Khadi and Village Industry clusters

• Coir Board – Coir based clusters

69. Consider the following statements

1. Five states shares border with Myanmar.

2. Six states shares border with Nepal

3. Ladakh region shares border with three countries.

Which of the following statements given above is/are correct?

(a) 1 and 3 only

(b) 2 only

(c) 1 and 2 only

(d) 3 only

Solution: D

• Arunachal Pradesh, Nagaland, Mizoram, Manipur shares border with Myanmar.

• Uttarkhand, Uttar Pradesh, Bihar, West Bengal, Sikkim shares border with Nepal.

• Ladakh shares border with China, Pakistan and Afghanistan

Page 63: SIMPLYFYING IAS EXAM PREPARATIONDAY – 41 (InstaTest-41) 1. Consider the following statements regarding Janaushadhi Sugam app 1. It has been launched by Ministry of Health and Family

INSTA 75 Days REVISION PLAN for Prelims 2020 - InstaTests

www.insightsonindia.com 61 Insights IAS

70. Consider the following statements regarding Permanent Court of Arbitration (PCA)

1. It aims to facilitate arbitration and other forms of dispute resolution between

states

2. It is a United Nations agency.

3. India is not a party to the convention.

Which of the statements given above is/are correct?

(a) 1 only

(b) 2 and 3 only

(c) 1 and 2 only

Page 64: SIMPLYFYING IAS EXAM PREPARATIONDAY – 41 (InstaTest-41) 1. Consider the following statements regarding Janaushadhi Sugam app 1. It has been launched by Ministry of Health and Family

INSTA 75 Days REVISION PLAN for Prelims 2020 - InstaTests

www.insightsonindia.com 62 Insights IAS

(d) 1, 2 and 3

Solution: A

Permanent Court of Arbitration (PCA)

• Intergovernmental organization

• Established in 1899 by Convention on the Pacific Settlement of disputes

• Headquarter — The Hague, Netherlands

• Membership- 121 states

• India is a party to the convention

• It is not a United Nations agency but PCA is an official United Nations Observer

• It is not a court “in the traditional sense” but provides services of arbitral tribunal to

resolve disputes that arise out of international agreements between member states,

international organizations or private parties

• The cases span a range of legal issues involving territorial and maritime boundaries,

sovereignty, human rights, international investment, and international and regional

trade

• All decisions, called “awards” are binding on all the parties in the dispute and have to

be carried out without delay.

• Established in 1899 to facilitate arbitration and other forms of dispute resolution

between states, the PCA has developed into a modern, multi-faceted arbitral

institution perfectly situated to meet the evolving dispute resolution needs of the

international community.

71. Special Economic Zones (SEZ) have been setup by the government to improve export

prospects of India. In this context, consider the following statements

1. It is a geographical region that has economic laws different than country’s typical

economic laws.

2. Only service sector companies are allowed to invest in SEZ’s.

Which of the statements given above is/are correct?

(a) 1 only

(b) 2 only

(c) Both 1 and 2

(d) Neither 1 nor 2

Solution: A

Page 65: SIMPLYFYING IAS EXAM PREPARATIONDAY – 41 (InstaTest-41) 1. Consider the following statements regarding Janaushadhi Sugam app 1. It has been launched by Ministry of Health and Family

INSTA 75 Days REVISION PLAN for Prelims 2020 - InstaTests

www.insightsonindia.com 63 Insights IAS

• Special economic zones (SEZs) in India are areas that offer incentives to resident

businesses. SEZs typically offer competitive infrastructure, duty free exports, tax

incentives, and other measures designed to make it easier to conduct business.

Accordingly, SEZs in India are a popular investment destination for many

multinationals, particularly exporters.

• After extensive consultations, the SEZ Act, 2005, supported by SEZ Rules, came into

effect on 10th February, 2006, providing for drastic simplification of procedures and

for single window clearance on matters relating to central as well as state

governments.

The main objectives of the SEZ Act are:

• generation of additional economic activity

• promotion of exports of goods and services

• promotion of investment from domestic and foreign sources

• creation of employment opportunities

• development of infrastructure facilities

• Both service sector and manufacturing industries are allowed to invest in SEZ’s.

http://sezindia.nic.in/cms/introduction.php

72. Consider the following statements regarding Farmer Produce Organizations (FPOs)

scheme

1. Small Farmers Agri-business Consortium (SFAC) is the sole implementing agency

of this scheme.

2. National Project Management Agency (NPMA) would be providing overall project

guidance, data compilation and maintenance through integrated portal and

Information management.

3. FPOs will be formed and promoted through Cluster Based Business Organizations

(CBBOs) engaged at the State/Cluster level by implementing agencies.

Which of the statements given above is/are correct?

(a) 1 and 2 only

(b) 2 only

(c) 3 only

(d) 2 and 3 only

Solution: D

Page 66: SIMPLYFYING IAS EXAM PREPARATIONDAY – 41 (InstaTest-41) 1. Consider the following statements regarding Janaushadhi Sugam app 1. It has been launched by Ministry of Health and Family

INSTA 75 Days REVISION PLAN for Prelims 2020 - InstaTests

www.insightsonindia.com 64 Insights IAS

• The Cabinet Committee on Economic Affairs, chaired by the Prime Minister Shri

Narendra Modi, has given its approval for 10,000 FPOs to be formed in five years

period from 2019-20 to 2023-24 to ensure economies of scale for farmers. Support to

each FPO be continued for 5 years from its year of inception.

Brief of the Scheme:

• A new Central Sector Scheme titled “Formation and Promotion of Farmer Produce

Organizations (FPOs)” to form and promote 10,000 new FPOs with a total budgetary

provision of Rs. 4496.00 crore for five years (2019-20 to 2023-24) with a further

committed liability of Rs. 2369.00 crore for period from 2024-25 to 2027-28 towards

handholding of each FPO for five years from its aggregation and formation.

• Initially there will be three implementing Agencies to form and promote FPOs, namely

Small Farmers Agri-business Consortium (SFAC), National Cooperative Development

Corporation (NCDC) and National Bank for Agriculture and Rural Development

(NABARD). States may also, if so desire, nominate their Implementing Agency in

consultation with DAC&FW.

• DAC&FW will allocate Cluster/States to Implementing Agencies which in turn will form

the Cluster Based Business Organization in the States.

• FPOs will be formed and promoted through Cluster Based Business Organizations

(CBBOs) engaged at the State/Cluster level by implementing agencies. The CBBOs will

have five categories of specialists from the domain of Crop husbandry, Agri marketing

/ Value addition and processing, Social mobilisation, Law & Accounts and IT/MIS.

These CBBOs will be platform for an end to end knowledge for all issues in FPO

promotion.

• There will be a National Project Management Agency (NPMA) at SFAC for providing

overall project guidance, data compilation and maintenance through integrated portal

and Information management and monitoring.

• Initially the minimum number of members in FPO will be 300 in plain area and 100 in

North East & hilly areas. However, DAC&FW may revise the minimum number of

membership based on experience/need with approval of Union Agriculture Minister.

• Priority will be given for formation of FPOs in aspirational districts in the country with

at least one FPO in each block of aspirational districts.

• FPOs will be promoted under “One District One Product” cluster to promote

specialization and better processing, marketing, branding & export by FPOs.

• There will be a provision of Equity Grant for strengthening equity base of FPOs.

• There will be a Credit Guarantee Fund of up to Rs. 1,000.00 crore in NABARD with

equal contribution by DAC&FW and NABARD and Credit Guarantee Fund of Rs.500.00

crore in NCDC with equal contribution by DAC&FW and NCDC for providing suitable

credit guarantee cover to accelerate flow of institutional credit to FPOs by minimizing

the risk of financial institutions for granting loan to FPOs.

• States/UTs will be allowed to avail loan at prescribed concessional rate of interest

under Agri-Market Infrastructure Fund (AMIF) approved for set up in NABARD for

developing agriculture marketing and allied infrastructure in GrAMs, by making

Page 67: SIMPLYFYING IAS EXAM PREPARATIONDAY – 41 (InstaTest-41) 1. Consider the following statements regarding Janaushadhi Sugam app 1. It has been launched by Ministry of Health and Family

INSTA 75 Days REVISION PLAN for Prelims 2020 - InstaTests

www.insightsonindia.com 65 Insights IAS

marketing & allied infrastructure including Common Facilitation Centre / Custom

Hiring Centre for FPOs as eligible category for providing assistance to States / UTs.

• Adequate training and handholding will be provided to FPOs. CBBOs will provide initial

training. Professional training of CEO / Board of Directors / Accountant of FPOs will be

provided in organizational training, resource planning, Accounting / management,

marketing, processing etc in reputed National / Regional training Institutes.

https://pib.gov.in/newsite/PrintRelease.aspx?relid=199421

73. Lighthouse India Initiative, sometime seen in the news, is related to

(a) Construction of lighthouses at strategic locations in India

(b) Lighting the houses of poor and marginalized sections of the society

(c) Knowledge dissemination programme driven by World Bank

(d) None of the statements (a), (b) and (c) are correct.

Solution: C

• The World Bank Group supports systematic efforts to create and curate knowledge

and know-how generated within India and disseminate it across the country as well as

to the outside world.

• Support for a Lighthouse India includes amplifying the country’s role as a leader on

key global development issues and leveraging its vast experience to provide for two-

way learning. The World Bank Group also supports sustained dialogue among policy

influencers and the private sector in the South Asia region to sharpen awareness and

strengthen coalitions for action on common development challenges within and

across countries.

https://www.worldbank.org/en/cpf/india/how-we-work/supporting-lighthouse-india

74. Consider the following statements regarding National Innovation Foundation

1. It is an autonomous body of the Department of Science and Technology.

2. It is an initiative to strengthen the grassroots technological innovations and

outstanding traditional knowledge.

Which of the statements given above is/are correct?

(a) 1 only

(b) 2 only

Page 68: SIMPLYFYING IAS EXAM PREPARATIONDAY – 41 (InstaTest-41) 1. Consider the following statements regarding Janaushadhi Sugam app 1. It has been launched by Ministry of Health and Family

INSTA 75 Days REVISION PLAN for Prelims 2020 - InstaTests

www.insightsonindia.com 66 Insights IAS

(c) Both 1 and 2

(d) Neither 1 nor 2

Solution: C

National Innovation Foundation

• NIF invite innovative citizens to participate in Challenge COVID-19 Competition (C3).

• The initiative will not only create awareness, but will intimately engage a wide cross-

section of society with diverse backgrounds in providing and implementing solutions.

About National Innovation Foundation (NIF) – India:

• It is an autonomous body of the Department of Science and Technology (DST),

Government of India. Set up in February 2000 at Ahmedabad, Gujarat to provide

institutional support for scouting, spawning, sustaining and scaling up the grassroots

innovations across the country. It is India’s national initiative to strengthen the

grassroots technological innovations and outstanding traditional knowledge. Its

mission is to help India become a creative and knowledge-based society by expanding

policy and institutional space for grassroots technological innovators.

Related key facts:

• The INSPIRE Award – MANAK (Million Minds Augmenting National Aspiration and

Knowledge) is being revamped and executed by Department of Science & Technology

and National Innovation Foundation-India to align it with the action plan for “Start-up

India” initiative.

• Micro Venture Innovation Fund (MVIF) at NIF, with support from Small Industries

Development Bank of India (SIDBI), has provided risk capital to 230 innovation based

enterprise projects, some of which are at different stages of incubation.

• Being organised since 2008, IGNITE is an annual competition for student’s ideas and

innovations conducted by NIF in partnership with the Central Board of Secondary

Education (CBSE).

• NIF has set up a Technology Business Incubator (NIF) – NIF Incubation and

Entrepreneurship Council (NIFientreC).

75. Consider the following statements regarding Belmont Forum

1. It is a high-level group of the world’s major and emerging funders of global

environmental change research and international science councils.

2. India is a member of Belmont Forum

3. Ministry of Environment, Forest and Climate Change represents India in the

Belmont Forum.

Page 69: SIMPLYFYING IAS EXAM PREPARATIONDAY – 41 (InstaTest-41) 1. Consider the following statements regarding Janaushadhi Sugam app 1. It has been launched by Ministry of Health and Family

INSTA 75 Days REVISION PLAN for Prelims 2020 - InstaTests

www.insightsonindia.com 67 Insights IAS

Which of the statements given above is/are correct?

(a) 1 and 2 only

(b) 2 and 3 only

(c) 1 only

(d) 1, 2 and 3

Solution: A

The Union Cabinet has given its approval for signing of the Collaborative Agreement with

French National Research Agency (ANR), France for supporting the Belmont Forum

Secretariat from January, 2015 to December, 2017 at a total estimated expenditure of Euro

40,000. The Cabinet also approved continued financial support to Belmont Forum Secretariat

beyond 2017.

What is Belmont Forum?

• The Belmont Forum, created in 2009, is a high-level group of the world’s major and

emerging funders of global environmental change research and international science

councils.

What it does?

• It provides an opportunity to identify study and deliver international environmental

research priorities, for the society, in an accelerated way through trans­national

research collaboration between natural and social scientists and alignment of

international resources

Members:

• India is a member of Belmont Forum, besides Australia, Brazil, Canada, European

Commission, France, Germany, Japan, Netherland, South Africa, UK and USA etc.

• Ministry of Earth Science (MoES), represents India in the Belmont Forum since 2012.

DAY – 44 (InstaTest-44)

76. Cyberdome Project, sometime seen in the news, is launched by which of the state

government?

(a) Karnataka

(b) Andhra Pradesh

Page 70: SIMPLYFYING IAS EXAM PREPARATIONDAY – 41 (InstaTest-41) 1. Consider the following statements regarding Janaushadhi Sugam app 1. It has been launched by Ministry of Health and Family

INSTA 75 Days REVISION PLAN for Prelims 2020 - InstaTests

www.insightsonindia.com 68 Insights IAS

(c) Kerala

(d) Tamil Nadu

Solution: C

Kerala Police counter online threats via an expert crew

• Cyberdome is based at an inconspicuous office on the Technopark campus here in

Thiruvananthapuram. It is the Kerala Police Department’s premier facility dedicated

to prevent cybercrime and mitigate cybersecurity threats to the State’s critical

information infrastructure.

• A motley crew of ethical hackers, expert coders, youth prodigies skilled in software,

law enforcers and civilian volunteers huddle behind flickering computer terminals at

the office, with most of them offering their services for free. They are the State law

enforcement’s first line of defence against a range of online threats.

• In March 2017, Cyberdome notched up a remarkable victory when it forewarned the

Kerala government of a possible ransomware attack.

77. Consider the following statements regarding Periodic Labour Force Survey (PLFS)

1. The Union Ministry of Statistics and Programme Implementation had constituted

PLFS under the chairmanship of Amitabh Kundu.

2. The unemployment rates in rural areas are higher than those in urban areas.

3. Among social groups, the highest unemployment rates are among the scheduled

castes.

Which of the statements given above is/are correct?

(a) 1 only

(b) 2 and 3 only

(c) 1 and 3 only

(d) 1, 2 and 3

Solution: A

Periodic Labour Force Survey (PLFS)

• The Union Ministry of Statistics and Programme Implementation had constituted

PLFS under the chairmanship of Amitabh Kundu.

Page 71: SIMPLYFYING IAS EXAM PREPARATIONDAY – 41 (InstaTest-41) 1. Consider the following statements regarding Janaushadhi Sugam app 1. It has been launched by Ministry of Health and Family

INSTA 75 Days REVISION PLAN for Prelims 2020 - InstaTests

www.insightsonindia.com 69 Insights IAS

• Key findings: The unemployment rate (UR) in both rural and urban India is at its

highest since 1972.

• In urban areas, the UR among men is more than twice and has increased twice among

women since 2011- 12.

• The unemployment rates in urban areas are higher than those in rural areas.

• In urban areas, the unemployment rates for females are higher than those for males.

• unemployment among rural not-literate females has reduced and among urban

females, the number of those who are literate up to primary-level jobs, is the same as

2011-12.

• Among social groups, the highest UR is among the ‘General’ or ‘Others’ category —

6.7 per cent.

• Among religious groups, Christians have the highest UR in both urban and rural areas.

78. Consider the following statements regarding Partial Credit Guarantee Scheme

1. It is a scheme initiated by Ministry of Micro, Small and Medium Enterprises.

2. It was initiated to make available collateral-free credit to the micro and small

enterprise sector.

Which of the statements given above is/are correct?

(a) 1 only

(b) 2 only

(c) Both 1 and 2

(d) Neither 1 nor 2

Solution: D

The proposed Government Guarantee support and resultant pool buyouts will help address

NBFCs/HFCs resolve their temporary liquidity or cash flow mismatch issues.

• This will also enable them to continue contributing to credit creation and providing

last mile lending to borrowers, thereby spurring economic growth.

Partial Credit Guarantee Scheme

• Partial Credit Guarantee Scheme for NBFCs gets cabinet nod.

• Key features of the scheme: It allows for purchase of high-rated pooled assets from

financially-sound non-banking financial companies (NBFCs) and housing finance

companies (HFCs) by public sector banks (PSBs).

Page 72: SIMPLYFYING IAS EXAM PREPARATIONDAY – 41 (InstaTest-41) 1. Consider the following statements regarding Janaushadhi Sugam app 1. It has been launched by Ministry of Health and Family

INSTA 75 Days REVISION PLAN for Prelims 2020 - InstaTests

www.insightsonindia.com 70 Insights IAS

• The scheme would cover NBFCs and HFCs that might have slipped into “Special

Mention Accounts (SMA)-0” category during the one-year period prior to August 1,

2018, and asset pools rated “BBB+” or higher.

• The window for one-time partial credit guarantee offered by the government would

remain open till June 30, 2020 or till such date by which Rs 1 lakh crore worth of assets

get purchased by the banks, whichever is earlier.

• The Finance Minister would have the power to extend the validity of the scheme by

up to three months by taking into account its progress.

Special Mention Accounts (SMA): The classification of Special Mention Accounts (SMA) was

introduced by the RBI in 2014, to identify those accounts that has the potential to become

an NPA/Stressed Asset.

• Special Mention Accounts are those assets/accounts that shows symptoms of bad

asset quality in the first 90 days itself.

The Special Mention Accounts are usually categorized in terms of duration.

• SMA – NF: Non-financial (NF) signals of stress

• SMA-0: Principal or interest payment not overdue for more than 30 days but account

showing signs of incipient stress.

• SMA- 1: Principal or interest payment overdue between 31-60days.

• SMA – 2: Principal or interest payment overdue between 61-90 days.

https://www.insightsonindia.com/2019/12/12/partial-credit-guarantee-scheme/

79. Consider the following statements regarding Insights Mission

1. It is a joint mission of NASA and Canadian Space Agency.

2. It is the first outer space robotic explorer to study in-depth the inner space of

Mars: its crust, mantle, and core.

Which of the statements given above is/are correct?

(a) 1 only

(b) 2 only

(c) Both 1 and 2

(d) Neither 1 nor 2

Solution: B

• InSight, short for Interior Exploration using Seismic Investigations, Geodesy and Heat

Transport, is a Mars lander designed to give the Red Planet its first thorough checkup

Page 73: SIMPLYFYING IAS EXAM PREPARATIONDAY – 41 (InstaTest-41) 1. Consider the following statements regarding Janaushadhi Sugam app 1. It has been launched by Ministry of Health and Family

INSTA 75 Days REVISION PLAN for Prelims 2020 - InstaTests

www.insightsonindia.com 71 Insights IAS

since it formed 4.5 billion years ago. It is the first outer space robotic explorer to study

in-depth the “inner space” of Mars: its crust, mantle, and core.

Interior of Mars

• Mars’ Interior: Artist’s rendition showing the inner structure of Mars. The topmost

layer is known as the crust, underneath it is the mantle, which rests on a solid inner

core.

• Studying Mars’ interior structure answers key questions about the early formation of

rocky planets in our inner solar system – Mercury, Venus, Earth, and Mars – more than

4 billion years ago, as well as rocky exoplanets. InSight also measures tectonic activity

and meteorite impacts on Mars today.

• The lander uses cutting edge instruments, to delve deep beneath the surface and seek

the fingerprints of the processes that formed the terrestrial planets. It does so by

measuring the planet’s “vital signs”: its “pulse” (seismology), “temperature” (heat

flow), and “reflexes” (precision tracking).

• This mission is part of NASA’s Discovery Program for highly focused science missions

that ask critical questions in solar system science.

https://mars.nasa.gov/insight/mission/overview/

80. Consider the following statements regarding External Benchmark Rates

1. The RBI has made it compulsory for banks to link their new floating rate home,

auto and MSME loans to an external benchmark.

2. Repo rate is one of the external benchmarks published by RBI.

3. At present, banks use Marginal Cost-based Lending Rate (MCLR) to arrive at their

lending rate.

Which of the statements given above is/are correct?

(a) 1 and 2 only

(b) 2 and 3 only

(c) 1 and 3 only

(d) 1, 2 and 3

Solution: C

External Benchmark Rates

• The RBI has made it compulsory for banks to link their new floating rate home, auto

and MSME loans to an external benchmark so that the borrowers can enjoy lower

rate of interest.

Page 74: SIMPLYFYING IAS EXAM PREPARATIONDAY – 41 (InstaTest-41) 1. Consider the following statements regarding Janaushadhi Sugam app 1. It has been launched by Ministry of Health and Family

INSTA 75 Days REVISION PLAN for Prelims 2020 - InstaTests

www.insightsonindia.com 72 Insights IAS

• Banks can choose from one of the four external benchmarks — repo rate, three-

month treasury bill yield, six-month treasury bill yield or any other benchmark

interest rate published by Financial Benchmarks India Private Ltd.

Background:

• Earlier interest rates on loans were linked to a bank’s marginal cost of fund-based

interest rate, known as the Marginal Cost of Lending Rate (MCLR).

• Existing loans and credit limits linked to the MCLR, base rate or Benchmark Prime

Lending Rate, would continue till repayment or renewal.

What is external benchmarking of loans?

• When you borrow money from a bank, be it for purchasing a house, car or for business

purposes, interest is levied based on certain methodologies approved by the Reserve

Bank of India (RBI). At present, banks use Marginal Cost-based Lending Rate (MCLR)

to arrive at their lending rate.

• Prior to this, it was the Base Rate method and the Benchmark Prime Lending Rate

(BPLR). These were all internal benchmarks. Banks have been allowed to use RBI’s

policy rate among other market-driven options to calculate lending rates.

81. Global Microscope on Financial Inclusion report is released by

(a) Oxfam International

(b) The Economist Intelligence Unit

(c) World Bank

(d) Organization for Economic Co-operation and Development

Solution: B

Global Microscope on Financial Inclusion report

• The Economist Intelligence Unit has released the 2019 edition of Global Microscope

on Financial Inclusion report.

• The 2019 edition of Global Microscope report features 11 new gender focussed

indicators that measure financial inclusion for both women as well as men.

About the report:

• Produced by Economist Intelligence Unit (EIU), the research and analysis division of

The Economist Group.

Page 75: SIMPLYFYING IAS EXAM PREPARATIONDAY – 41 (InstaTest-41) 1. Consider the following statements regarding Janaushadhi Sugam app 1. It has been launched by Ministry of Health and Family

INSTA 75 Days REVISION PLAN for Prelims 2020 - InstaTests

www.insightsonindia.com 73 Insights IAS

• The Microscope report was first published in 2007 and was originally developed for

countries in Latin American and Caribbean regions but in 2009 it was expanded into a

global study.

Five parameters across which countries are assessed:

1. Government and Policy Support.

2. Products and Outlets.

3. Stability and Integrity.

4. Consumer Protection.

5. Infrastructure.

82. Consider the following statements regarding 9th schedule of Indian Constitution

1. It contains a list of central and state laws which cannot be challenged in courts

2. First constitutional amendment added 9th schedule into the constitution.

3. It is not subjected to judicial review

Which of the statements given above is/are correct?

(a) 1 and 3 only

(b) 2 only

(c) 1 and 2 only

(d) 1, 2 and 3

Solution: C

9th Schedule contains a list of central and state laws which cannot be challenged in courts. It

was added with the 1st amendment in 1951.

• In IR Coelho versus State of Tamil Nadu case, Supreme Court held that laws placed in

the 9th Schedule were open to judicial scrutiny

• SC laid down dual test to examine the validity of a law placed in the Ninth Schedule

i.e. Whether it violates any fundamental right and if yes whether the violation also

damages or destroys the basic structure.

83. Consider the following statements regarding INSTEX

1. It is a project of the governments of Russia, China and United Kingdom.

2. It is a payment mechanism to secure trade with Iran.

3. Its mission is to facilitate non-USD transactions and non-SWIFT to avoid breaking

U.S. sanctions.

Page 76: SIMPLYFYING IAS EXAM PREPARATIONDAY – 41 (InstaTest-41) 1. Consider the following statements regarding Janaushadhi Sugam app 1. It has been launched by Ministry of Health and Family

INSTA 75 Days REVISION PLAN for Prelims 2020 - InstaTests

www.insightsonindia.com 74 Insights IAS

Which of the statements given above is/are correct?

(a) 1 and 2 only

(b) 2 and 3 only

(c) 1 and 3 only

(d) 1, 2 and 3

Solution: B

INSTEX – Instrument In Support Of Trade Exchanges

Six Countries – Belgium, Denmark, Finland, the Netherlands, Norway and Sweden have

recently joined INSTEX.

What is it?

• It is a payment mechanism being setup by the European Union to secure trade with

Iran and skirt US sanctions after Washington pulled out of the landmark nuclear deal

last May.

• Its mission is to facilitate non-USD transactions and non-SWIFT to avoid breaking U.S.

sanctions.

• It is registered at Paris with an initial 3,000 Euros in the capital and a supervisory board

with members from France and Germany and chaired by the UK.

• It is a project of the governments of France, Germany and United Kingdom and will

receive the formal endorsement of all 28 EU members.

Key features of INSTEX:

• It will allow trade between the EU and Iran without relying on direct financial

transactions.

• It will initially be used for non-sanctionable trade, including humanitarian goods such

as medicine, food and medical devices.

• This mechanism is the first concrete step by the EU to counter Trump’s unilateral

decision to withdraw from the nuclear deal.

84. Consider the following statements regarding Development Banks

1. They are financial institutions that provide long-term credit for capital-intensive

investments spread over a long period and yielding low rates of return.

2. Industrial Finance Corporation is the first industrial development bank.

3. IDBI was set up as an apex body of all development finance institutions.

Which of the statements given above is/are correct?

Page 77: SIMPLYFYING IAS EXAM PREPARATIONDAY – 41 (InstaTest-41) 1. Consider the following statements regarding Janaushadhi Sugam app 1. It has been launched by Ministry of Health and Family

INSTA 75 Days REVISION PLAN for Prelims 2020 - InstaTests

www.insightsonindia.com 75 Insights IAS

(a) 1 and 2 only

(b) 2 and 3 only

(c) 1 and 3 only

(d) 1, 2 and 3

Solution: D

Development Banks

• In order to improve access to long-term finance, the government has proposed to

establish an organisation to provide credit enhancement for infrastructure and

housing projects, particularly in the context of India now not having a development

bank.

What are development banks?

• They are financial institutions that provide long-term credit for capital-intensive

investments spread over a long period and yielding low rates of return, such as urban

infrastructure, mining and heavy industry, and irrigation systems.

• Development banks are also known as term-lending institutions or development

finance institutions.

Features of development banks:

• Such banks often lend at low and stable rates of interest to promote long-term

investments with considerable social benefits.

• Fund generation: To lend for long term, development banks require correspondingly

long-term sources of finance, usually obtained by issuing long-dated securities in

capital market, subscribed by long-term savings institutions such as pension and life

insurance funds and post office deposits.

• Support by the government: Considering the social benefits of such investments, and

uncertainties associated with them, development banks are often supported by

governments or international institutions.

• Such support can be in the form of tax incentives and administrative mandates for

private sector banks and financial institutions to invest in securities issued by

development banks.

Following foregoing precepts, IFCI, previously the Industrial Finance Corporation of India,

was set up in 1949. This was probably India’s first development bank for financing industrial

investments.

• Industrial Finance Corporation is the first industrial development bank set up by the

Government of India in July 1948.

Page 78: SIMPLYFYING IAS EXAM PREPARATIONDAY – 41 (InstaTest-41) 1. Consider the following statements regarding Janaushadhi Sugam app 1. It has been launched by Ministry of Health and Family

INSTA 75 Days REVISION PLAN for Prelims 2020 - InstaTests

www.insightsonindia.com 76 Insights IAS

• In 1955, the World Bank prompted the Industrial Credit and Investment Corporation

of India (ICICI) — the parent of the largest private commercial bank in India today,

ICICI Bank — as a collaborative effort between the government with majority equity

holding and India’s leading industrialists with nominal equity ownership to finance

modern and relatively large private corporate enterprises.

• In 1964, IDBI was set up as an apex body of all development finance institutions.

85. Emissions Gap report, sometime seen in the news, is released by which of the following

organisation?

(a) The Intergovernmental Panel on Climate Change (IPCC)

(b) United Nations Environment Programme.

(c) World Meteorological Organisation.

(d) None of the above

Solution: B

United Nations Environment Programme (UNEP) released ‘Emissions Gap Report’, according

to which India is the fourth-largest emitter of Green House Gases (GHGs).

• According to the report, unless global greenhouse gas emissions fall by 7.6 per cent

each year, the world will fail to meet the 1.5°C temperature goal of the Paris

Agreement.

• The top four emitters (China, USA, EU and India) contribute to over 55 per cent of the

total emissions over the last decade, excluding emissions from land-use change such

as deforestation, the report states.

• If land-use change emissions were included, the rankings would change, with Brazil

likely to be the largest emitter.

• Largest emitting sectors are Energy > Industry > Forestry > Transport > Agriculture.

https://www.unenvironment.org/resources/emissions-gap-report-2019

86. Consider the following statements regarding L2Pro India

1. It is an initiative of Ministry of Micro, Small and Medium Enterprises.

2. It is an e-learning platform which will aid and enable youth, innovators,

entrepreneurs and small and medium industries (SMEs) in understanding IPRs.

Which of the statements given above is/are correct?

(a) 1 only

Page 79: SIMPLYFYING IAS EXAM PREPARATIONDAY – 41 (InstaTest-41) 1. Consider the following statements regarding Janaushadhi Sugam app 1. It has been launched by Ministry of Health and Family

INSTA 75 Days REVISION PLAN for Prelims 2020 - InstaTests

www.insightsonindia.com 77 Insights IAS

(b) 2 only

(c) Both 1 and 2

(d) Neither 1 nor 2

Solution: B

• Secretary Department for Promotion of Industry and Internal Trade (DPIIT),

Guruprasad Mohapatra launched the website and mobile application [Learn to

Protect, Secure and Maximize Your Innovation] on Intellectual Property Rights (IPRs)

today in New Delhi. The website and app has been developed by Cell for IPR Promotion

and Management (CIPAM)-DPIIT in collaboration with Qualcomm and National Law

University (NLU), Delhi.

• The modules of this e-learning platform [L2Pro India IP e-learning Platform and the

L2Pro India Mobile App] will aid and enable youth, innovators, entrepreneurs and

small and medium industries (SMEs) in understanding IPRs for their ownership and

protection, integrate IP into business models and obtain value for their R&D efforts.

• The L2Pro has been successfully implemented in Germany, United Kingdom, Italy and

France, benefiting immensely from close collaboration with respective IP

organizations and public research institutions. The learning app has been customized

for India in order to ensure that innovation which is fundamental to startups are

protected, managed and commercialised.

87. Consider the following statements regarding the GDP calculation methodology in India

1. The Base Year for calculation of GDP was revised from 2004–05 to 2014–15.

2. The methodology of calculating the National Accounts has been revised in line

with the requirements of the System of National Accounts (SNA)-2008, an

internationally accepted standard.

Which of the statements given above is/are correct?

(a) 1 only

(b) 2 only

(c) Both 1 and 2

(d) Neither 1 nor 2

Solution: B

Page 80: SIMPLYFYING IAS EXAM PREPARATIONDAY – 41 (InstaTest-41) 1. Consider the following statements regarding Janaushadhi Sugam app 1. It has been launched by Ministry of Health and Family

INSTA 75 Days REVISION PLAN for Prelims 2020 - InstaTests

www.insightsonindia.com 78 Insights IAS

The Central Statistics Office (CSO), in January 2015, released the new and revised data of

National Accounts, effecting two changes:

• The Base Year was revised from 2004–05 to 2011–12. This was done in accordance

with the recommendation of the National Statistical Commission (NSC), which had

advised to revise the base year of all economic indices every five years.

• This time, the methodology of calculating the National Accounts has also been revised

in line with the requirements of the System of National Accounts (SNA)-2008, an

internationally accepted standard.

88. Consider the following statements regarding National Green Tribunal (NGT)

1. NGT was established under Environment Protection Act, 1986.

2. The NGT has the power to hear all civil and criminal cases relating to

environmental issues.

3. The NGT is bound by the procedure laid down under the Code of Civil Procedure,

1908.

Which of the statements given above is/are correct?

(a) 1 only

(b) 2 and 3 only

(c) 3 only

(d) None

Solution: D

The National Green Tribunal has been established on 18.10.2010 under the National Green

Tribunal Act 2010 for effective and expeditious disposal of cases relating to environmental

protection and conservation of forests and other natural resources including enforcement of

any legal right relating to environment and giving relief and compensation for damages to

persons and property and for matters connected therewith or incidental thereto.

The NGT has the power to hear all civil cases relating to environmental issues and questions

that are linked to the implementation of laws listed in Schedule I of the NGT Act. These

include the following:

• The Water (Prevention and Control of Pollution) Act, 1974;

• The Water (Prevention and Control of Pollution) Cess Act, 1977;

• The Forest (Conservation) Act, 1980;

• The Air (Prevention and Control of Pollution) Act, 1981;

• The Environment (Protection) Act, 1986;

Page 81: SIMPLYFYING IAS EXAM PREPARATIONDAY – 41 (InstaTest-41) 1. Consider the following statements regarding Janaushadhi Sugam app 1. It has been launched by Ministry of Health and Family

INSTA 75 Days REVISION PLAN for Prelims 2020 - InstaTests

www.insightsonindia.com 79 Insights IAS

• The Public Liability Insurance Act, 1991;

• The Biological Diversity Act, 2002.

This means that any violations pertaining only to these laws, or any order / decision taken by

the Government under these laws can be challenged before the NGT. Importantly, the NGT

has not been vested with powers to hear any matter relating to the Wildlife (Protection)

Act, 1972, the Indian Forest Act, 1927 and various laws enacted by States relating to forests,

tree preservation etc. Therefore, specific and substantial issues related to these laws cannot

be raised before the NGT.

The NGT is not bound by the procedure laid down under the Code of Civil Procedure, 1908.

89. Which of the following is/are the four pillars of Gross National Happiness?

1. Good governance

2. Standard of Living

3. Environmental conservation

4. Preservation and promotion of culture

Select the correct answer using the code given below

(a) 1, 3 and 4 only

(b) 2, and 4 only

(c) 1 and 3 only

(d) 1, 2, 3 and 4

Solution: A

What Is Gross National Happiness (GNH)?

• Gross national happiness (GNH) is a measure of economic and moral progress that

the king of the Himalayan country of Bhutan introduced in the 1970s as an alternative

to gross domestic product. Rather than focusing strictly on quantitative economic

measures, gross national happiness takes into account an evolving mix of quality-of-

life factors.

• The kingdom of Bhutan’s first legal code, written at the time of unification in 1729,

stated that “if the government cannot create happiness for its people, there is no

purpose for the government.”

• The “four pillars” of GNH are good governance, sustainable development,

preservation and promotion of culture, and environmental conservation.

Page 82: SIMPLYFYING IAS EXAM PREPARATIONDAY – 41 (InstaTest-41) 1. Consider the following statements regarding Janaushadhi Sugam app 1. It has been launched by Ministry of Health and Family

INSTA 75 Days REVISION PLAN for Prelims 2020 - InstaTests

www.insightsonindia.com 80 Insights IAS

90. Consider the following statements regarding Non-Banking Financial Company (NBFC)

1. NBFCs cannot accept demand deposits.

2. NBFCs form the part of the payment and settlement system.

3. They cannot issue cheques drawn on itself.

Which of the statements given above is/are correct?

(a) 1, 2 and 3

(b) 1 and 3 only

(c) 2 and 3 only

(d) None

Solution: B

A Non-Banking Financial Company (NBFC) is a company registered under the Companies Act,

1956 engaged in the business of loans and advances, acquisition of

shares/stocks/bonds/debentures/securities issued by Government or local authority or other

marketable securities of a like nature, leasing, hire-purchase, insurance business, chit

business.

• But it does not include any institution whose principal business is that of agriculture

activity, industrial activity, purchase or sale of any goods (other than securities) or

providing any services and sale/purchase/construction of immovable property.

• A non-banking institution which is a company and has principal business of receiving

deposits under any scheme or arrangement in one lump sum or in installments by way

of contributions or in any other manner, is also a non-banking financial company

(Residuary non-banking company).

NBFCs lend and make investments and hence their activities are akin to that of banks;

however, there are a few differences as given below:

• NBFC cannot accept demand deposits;

• NBFCs do not form part of the payment and settlement system and cannot issue

cheques drawn on itself;

• deposit insurance facility of Deposit Insurance and Credit Guarantee Corporation is

not available to depositors of NBFCs, unlike in case of banks.

91. Consider the following statements regarding Nirbhaya Fund

1. It was created by the Finance Ministry

2. It is a non-lapsable corpus fund which will be utilized for upholding safety and

dignity of women.

Page 83: SIMPLYFYING IAS EXAM PREPARATIONDAY – 41 (InstaTest-41) 1. Consider the following statements regarding Janaushadhi Sugam app 1. It has been launched by Ministry of Health and Family

INSTA 75 Days REVISION PLAN for Prelims 2020 - InstaTests

www.insightsonindia.com 81 Insights IAS

3. The Fund is administered by Department of Economic Affairs of the finance

ministry.

Which of the statements given above is/are correct?

(a) 1 only

(b) 1 and 3 only

(c) 2 and 3 only

(d) 1, 2 and 3

Solution: D

About Nirbhaya fund:

• The Rs 1,000 crore Nirbhaya Fund was announced in Union Budget 2013 by the

Finance Ministry.

• The corpus was to be utilised for upholding safety and dignity of women.

• Ministry of Women and Child Development apart from several other concerned

ministries were authorised to work out details of structure, scope and application of

this fund.

• The Fund is administered by Department of Economic Affairs of the finance ministry.

• It is a non-lapsable corpus fund.

• It is being monitored by Empowered Committee of Officers under Secretary of

Women and Child Development Ministry.

92. Consider the following statements regarding the Bombay plan

1. The Bombay Plan was prepared by a cross-section of India’s leading capitalists.

2. It was against the idea of the state to play an active role in the economy.

3. It planned for rapid industrialization with emphasis on heavy capital goods and

basic industries.

Which of the statements given above is/are correct?

(a) 1 and 2 only

(b) 2 and 3 only

(c) 1 and 3 only

(d) 1, 2 and 3

Solution: C

Page 84: SIMPLYFYING IAS EXAM PREPARATIONDAY – 41 (InstaTest-41) 1. Consider the following statements regarding Janaushadhi Sugam app 1. It has been launched by Ministry of Health and Family

INSTA 75 Days REVISION PLAN for Prelims 2020 - InstaTests

www.insightsonindia.com 82 Insights IAS

The Bombay plan

• The Bombay Plan was the popular title of ‘A Plan of Economic Development for

India’, which was prepared by a cross-section of India’s leading capitalists. The eight

capitalists involved in this plan were Purshotamdas Thakurdas, J.R.D. Tata, G.D. Birla,

Lala Sri Ram, Kasturbhai Lalbhai, A.D. Shroff, Avdeshir Dalal and John Mathai. The

Plan was published in 1944–45.

• The popular sentiments regarding the need of planning and criss-cross of

memberships between the NPC and the Bombay Plan club made possible some clear-

cut agreements between these two major plans.

• A basic agreement on the issue of the agrarian restructuring—abolition of all

intermediaries (i.e., zamindari abolition), minimum wages, guarantee of minimum or

fair prices for agricultural products, cooperatives, credit and marketing supports.

• Taking clues from the Soviet Planning, the NPC and the Bombay Plan both were in

favour of a simultaneous development of the essential consumer goods industries,

but as a low-key affair.

• Agreement on rapid industrialization for which both the plans agreed upon an

emphasis on heavy capital goods and basic industries

• Both the plans wanted the state to play an active role in the economy through

planning, controlling and overseeing the different areas of the economy, i.e., trade,

industry and banking, through state ownership (public sector) or through direct and

extensive control over them.

93. Statutory Liquidity Ratio can be maintained with which of the following assets

1. Cash

2. Gold

3. Treasury Bills

4. State Development Loans (SDLs)

Select the correct answer using the code given below:

(a) 1, 2 and 3 only

(b) 2, 3 and 4 only

(c) 1, 3 and 4 only

(d) 1, 2, 3 and 4

Solution: D

Page 85: SIMPLYFYING IAS EXAM PREPARATIONDAY – 41 (InstaTest-41) 1. Consider the following statements regarding Janaushadhi Sugam app 1. It has been launched by Ministry of Health and Family

INSTA 75 Days REVISION PLAN for Prelims 2020 - InstaTests

www.insightsonindia.com 83 Insights IAS

SLR assets shall be maintained by:

Scheduled Commercial Banks (Including Regional Rural Banks), Local Area Banks, Small

Finance Banks and Payments Banks, as –

(a) cash; or

(b) gold as defined in Section 5(g) of the Banking Regulation Act, 1949 (10 of 1949)

valued at a price not exceeding the current market price: or

(c) unencumbered investment in any of the following instruments [hereinafter

referred to as Statutory Liquidity Ratio securities (“SLR securities”)], namely:

1. Dated securities of the Government of India issued from time to time under

the Market Borrowing Programme and the Market Stabilization Scheme ;

or

2. Treasury Bills of the Government of India; or

3. State Development Loans (SDLs) of the State Governments issued from

time to time under the market borrowing programme:

(d) the deposit and unencumbered approved securities required, under sub-

section (2) of section 11 of the Banking Regulation Act, 1949(10 of 1949), to be

made with the Reserve Bank by a banking company incorporated outside India;

(e) any balance maintained by a scheduled bank with the Reserve Bank in excess

of the balance required to be maintained by it under section 42 of the Reserve

Bank of India Act,1934 (2 of 1934);

94. Consider the following statements regarding Krishi Kisan App

1. It will provide farmers the information of best demonstration of high-yielding

crops and seeds in their nearby area.

2. Any farmer with high quality of crops can utilize this platform to demonstrate best

practices of cultivation to other farmers

3. It will give weather forecast message to farmers.

Which of the statements given above is/are correct?

(a) 3 only

(b) 1 and 2 only

(c) 1 and 3 only

(d) 1, 2 and 3

Solution: D

Page 86: SIMPLYFYING IAS EXAM PREPARATIONDAY – 41 (InstaTest-41) 1. Consider the following statements regarding Janaushadhi Sugam app 1. It has been launched by Ministry of Health and Family

INSTA 75 Days REVISION PLAN for Prelims 2020 - InstaTests

www.insightsonindia.com 84 Insights IAS

• Central government has launched Krishi Kisan Mobile App for farmers on 24

September 2019. This mobile application will provide farmers the information of best

demonstration of high yielding crops and seeds in their nearby areas.

• Any farmer with high quality of crops can utilize this Krishi Kisan App platform. This

app will enable farmers to demonstrate best practices of cultivation to other farmers

so that this will help other farmers also to adopt these methods.

• Krishi Kisan app will also help in geo-tagging and geo-fencing of crop and give weather

forecast message to farmers.

• Every farmer can now utilize services available at Krishi Kisan Mobile App for the

betterment of their cultivation methods. This will ensure higher agricultural

productivity for farmers & will raise their income. Farmers can take benefits of field

demonstration of new farm technologies, seed hubs and weather advisories.

Krishi Kisan App Benefits & Implementation:

Crops Division of DAC&FW is undertaking the following activities through various Crops

Development Programmes:-

• Cluster demonstrations on pulses by KVKs, ICAR

• Cluster demonstrations by state governments

• Front Line Demonstrations (FLDs) conducted by ICAR Institutions on rice, wheat,

pulses, coarse-cereals and nutri-cereals

• Minikit-demonstrations on farmers field

• Seed production by seed hub centres of pulses and nutri-cereals

Krishi Kisan Mobile app has been developed to help farmers by quickly providing them with

necessary information. With just one click of a button, farmers can get the information on

demonstration in nearest area and Seed Mini kit distribution in their area.

95. Consider the following statements regarding Standing Deposit Facility (SDF)

1. It is a collateral free liquidity absorption mechanism that aims to absorb liquidity

from the commercial banking system into the RBI.

2. It was introduced in Budget 2020-2021.

Which of the statements given above is/are correct?

(a) 1 only

(b) 2 only

(c) Both 1 and 2

(d) Neither 1 nor 2

Solution: A

Page 87: SIMPLYFYING IAS EXAM PREPARATIONDAY – 41 (InstaTest-41) 1. Consider the following statements regarding Janaushadhi Sugam app 1. It has been launched by Ministry of Health and Family

INSTA 75 Days REVISION PLAN for Prelims 2020 - InstaTests

www.insightsonindia.com 85 Insights IAS

• The Standing Deposit Facility, proposed to be introduced by the RBI, is a collateral

free liquidity absorption mechanism that aims to absorb liquidity from the

commercial banking system into the RBI. Government in the Budget’s (2018) Finance

Act included a provision for the introduction of the Standing Deposit Facility (SDF).

What is Standing Deposit Facility (SDF)?

• Standing Deposit Facility allows the RBI to absorb liquidity (deposit) from commercial

banks without giving government securities in return to the banks. In the present

situation, the main arrangement for the RBI to absorb excess money with the banking

system is the famous reverse repo mechanism. Under reverse repo (which is a part of

the Liquidity Adjustment Facility), banks will get government securities in return when

they give excess cash to the RBI. An interest rate of reverse repo rate is also provided

to banks.

96. Consider the following statements regarding ‘UMMID’ initiative

1. It has been launched by Ministry of Health and Family Welfare.

2. It aims to create awareness about genetic disorders amongst clinicians and

establish molecular diagnostics in hospitals.

Which of the statements given above is/are correct?

(a) 1 only

(b) 2 only

(c) Both 1 and 2

(d) Neither 1 nor 2

Solution: B

• Government launches ‘UMMID’ initiative to tackle inherited genetic diseases of new

born babies. Department of Biotechnology has started the UMMID Initiative which is

designed on the concept of ‘Prevention is better than Cure’.

UMMID initiative aims to:

• UMMID aims to create awareness about genetic disorders amongst clinicians and

establish molecular diagnostics in hospitals so that the fruits of developments in

medical genetics reach the patients in India.

• Establish NIDAN (National Inherited Diseases Administration) Kendras to provide

counselling, prenatal testing and diagnosis, management, and multidisciplinary care

in Government Hospitals wherein the influx of patients is more.

• Produce skilled clinicians in Human Genetics.

Page 88: SIMPLYFYING IAS EXAM PREPARATIONDAY – 41 (InstaTest-41) 1. Consider the following statements regarding Janaushadhi Sugam app 1. It has been launched by Ministry of Health and Family

INSTA 75 Days REVISION PLAN for Prelims 2020 - InstaTests

www.insightsonindia.com 86 Insights IAS

• Undertake screening of pregnant women and new born babies for inherited genetic

diseases in hospitals at aspirational districts.

97. Which of the following forms the part of Priority Sector Lending

1. Micro, Small and Medium Enterprises

2. Export Credit

3. Renewable Energy

4. Education

Select the correct answer using the code given below:

(a) 1, 2 and 3 only

(b) 2, 3 and 4 only

(c) 1, 3 and 4 only

(d) 1, 2, 3 and 4

Solution: D

Page 89: SIMPLYFYING IAS EXAM PREPARATIONDAY – 41 (InstaTest-41) 1. Consider the following statements regarding Janaushadhi Sugam app 1. It has been launched by Ministry of Health and Family

INSTA 75 Days REVISION PLAN for Prelims 2020 - InstaTests

www.insightsonindia.com 87 Insights IAS

Priority Sector Lending

Priority Sector includes the following categories:

(i) Agriculture

(ii) Micro, Small and Medium Enterprises

(iii) Export Credit

(iv) Education

(v) Housing

(vi) Social Infrastructure

(vii) Renewable Energy

(viii) Others

What are the Targets and Sub-targets for banks under priority sector?

The targets and sub-targets for banks under priority sector are as follows:

Categories Domestic scheduled commercial banks (excluding Regional Rural Banks and Small Finance Banks) and Foreign banks with 20 branches and above

Foreign banks with less than 20

branches

Total Priority Sector 40 per cent of Adjusted Net Bank Credit or Credit Equivalent Amount of Off-Balance Sheet Exposure, whichever is higher.

40 per cent of Adjusted Net Bank

Credit or Credit Equivalent Amount

of Off-Balance Sheet Exposure,

whichever is higher, to be achieved in a phased manner by

2020.

Agriculture* 18 per cent of ANBC or Credit Equivalent Amount of Off-Balance Sheet Exposure, whichever is higher. Within the 18 percent target for agriculture, a target of 8 percent of ANBC or Credit Equivalent Amount of Off-Balance Sheet Exposure, whichever is higher is prescribed for Small and Marginal Farmers.

Not applicable

Micro Enterprises 7.5 percent of ANBC or Credit Equivalent Amount of Off-Balance Sheet Exposure, whichever is higher.

Not applicable

Page 90: SIMPLYFYING IAS EXAM PREPARATIONDAY – 41 (InstaTest-41) 1. Consider the following statements regarding Janaushadhi Sugam app 1. It has been launched by Ministry of Health and Family

INSTA 75 Days REVISION PLAN for Prelims 2020 - InstaTests

www.insightsonindia.com 88 Insights IAS

Advances to Weaker Sections

10 percent of ANBC or Credit Equivalent Amount of Off-Balance Sheet Exposure, whichever is higher

Not applicable

*Domestic banks have been directed to ensure that their overall direct lending to non-

corporate farmers does not fall below the system-wide average of the last three years

achievement.

98. Consider the following statements regarding All India Survey on Higher Education

(AISHE)

1. It has been released by Ministry of Human Resource Development

2. The survey covers all the Institutions in the country engaged in higher education.

3. According to recent findings gender gap is narrowing with an improvement in

female enrollment ratio.

Which of the statements given above is/are correct?

(a) 1 only

(b) 2 and 3 only

(c) 1 and 2 only

(d) 1, 2 and 3

Solution: D

All India Survey on Higher Education (AISHE)

• All India Survey on Higher Education (AISHE) report for 2018-19 was recently released

by the HRD ministry.

• The survey, undertaken as an annual, web-based, pan-India exercise on the status of

Higher Education since 2010-11.

• The survey covers all the Institutions in the country engaged in higher education.

• Data is being collected on several parameters such as teachers, student enrolment,

programmes, examination results, education finance, and infrastructure.

• Indicators of educational development such as Institution Density, Gross Enrolment

Ratio, Pupil-teacher ratio, Gender Parity Index, Per Student Expenditure will also be

calculated from the data collected through AISHE.

Key findings:

• General trend indicates gender gap is narrowing with an improvement in female

enrollment ratio from 47.6% in 2017-18 to 48.6% in 2018-19.

Page 91: SIMPLYFYING IAS EXAM PREPARATIONDAY – 41 (InstaTest-41) 1. Consider the following statements regarding Janaushadhi Sugam app 1. It has been launched by Ministry of Health and Family

INSTA 75 Days REVISION PLAN for Prelims 2020 - InstaTests

www.insightsonindia.com 89 Insights IAS

• More Girls in two states: In Uttar Pradesh and Karnataka, there are now more

females in the age group of 18-23 enrolling for higher education than male students.

• The gross enrolment ratio (GER) increased marginally – from 25.8 in 2017-18 to 26.3

in 2018-19.

• In absolute terms, enrolment increased from 3.66 crore to 3.74 crore students in the

same period. GER for SCs has also shown a growth from 21.8 to 23.0 and STs from

15.9 to 17.2.

• Number of universities has grown from 903 in 2017-18 to 993 in 2018-19 and total

HEIs (higher educational institutions) from 49,964 to 51,649 in the same period.

• Number of faculty has also increased from 13.88 lakh to 14.16 lakh.

• Preferred stream at PG level: While one-third of undergraduate students are enrolled

in humanities, management seems to be a preferred stream at the postgraduate (PG)

level. Science and engineering technology registered relatively more enrolment in M.

Phil and Ph.D programmes.

• Preferred stream at the UG level, 35.9% of the total enrolment was in arts/

humanities/social science, just 16.5% students are pursuing science, followed by

commerce with 14.1%. Engineering is the fourth choice.

99. Consider the following statements regarding Regional Rural Banks (RRBs)

1. They are financial institutions which ensure adequate credit for agriculture and

other rural sectors.

2. Regional Rural Banks were set up on the basis of the recommendations of the

Narasimham Working Group (1975).

3. The equity of a regional rural bank is held by the Central Government and State

Government on 50:50 ratio basis.

Which of the statements given above is/are correct?

(a) 1 and 2 only

(b) 2 and 3 only

(c) 1 only

(d) 1, 2 and 3

Solution: A

Regional Rural Banks (RRBs) are financial institutions which ensure adequate credit for

agriculture and other rural sectors. Regional Rural Banks were set up on the basis of the

recommendations of the Narasimham Working Group (1975), and after the legislations of

the Regional Rural Banks Act, 1976. The first Regional Rural Bank “Prathama Grameen

Bank” was set up on October 2, 1975. At present there are 82 RRBs in India.

Page 92: SIMPLYFYING IAS EXAM PREPARATIONDAY – 41 (InstaTest-41) 1. Consider the following statements regarding Janaushadhi Sugam app 1. It has been launched by Ministry of Health and Family

INSTA 75 Days REVISION PLAN for Prelims 2020 - InstaTests

www.insightsonindia.com 90 Insights IAS

• The equity of a regional rural bank is held by the Central Government, concerned State

Government and the Sponsor Bank in the proportion of 50:15:35. The RRBs combine

the characteristics of a cooperative in terms of the familiarity of the rural problems

and a commercial bank in terms of its professionalism and ability to mobilise financial

resources. Each RRB operates within the local limits as notified by Government. The

main objectives of RRB’s are to provide credit and other facilities‚ especially to the

small and marginal farmers‚ agricultural labourers artisans and small entrepreneurs in

rural areas with the objective of bridging the credit gap in rural areas, checking the

outflow of rural deposits to urban areas and reduce regional imbalances and increase

rural employment generation.

• The RRB’s have also been brought under the ambit of priority sector lending on par

with the commercial banks. Priority sector lending has been devised so that assistance

from the banking system flowed in an increasing measure to the vital sectors of the

economy and according to national priorities. Sectors like agriculture, small business,

housing, retail trade, education are categorised as priority sector by Reserve Bank of

India and a stipulated amount has to be lent to these sectors by the banks. As per the

guidelines, domestic banks have to ensure that forty percent of their advances are

accounted for the priority sector. Within the 40% priority target, 25% should go to

weaker section or 10% of their total advances should go to the weaker section

.Weaker sections, under priority sector lending purposes, include scheduled castes,

scheduled tribes, small and marginal farmers, artisans and self help groups.

• RRBs have been finding it difficult to maintain their minimum Capital to Risk weighted

Assets Ratio (CRAR) at the stipulated 9%.With a view to bring the CRAR of RRBs to at

least 9 percent, the Dr. K. C. Chakrabarty Committee recommended recapitalisation

support to the extent of Rs.2,200 crore to 40 RRBs in 21 States. The recapitalisation

process started in 2010-11.

100. Which of the following methods is/are used to tackle arsenic contamination in water?

1. Adsorption

2. Ion-Exchange

3. Electro kinetic treatment

4. Oxidation

Select the correct answer using the code given below

(a) 3 and 4 only

(b) 1, 2 and 4 only

(c) 1, 2, 3 and 4

(d) 1, 2 and 3 only

Solution: C

Page 93: SIMPLYFYING IAS EXAM PREPARATIONDAY – 41 (InstaTest-41) 1. Consider the following statements regarding Janaushadhi Sugam app 1. It has been launched by Ministry of Health and Family

INSTA 75 Days REVISION PLAN for Prelims 2020 - InstaTests

www.insightsonindia.com 91 Insights IAS

• Report on Groundwater Arsenic Contamination in India is released by Central Ground

Water Board (CGWB).

• 21 states across the country have pockets with arsenic levels higher than the Bureau

of Indian Standards (BIS) stipulated permissible limit of 0.01 miligram per litre (mg/l)

• States along the Ganga-Brahmaputra-Meghna (GBM) river basin Uttar Pradesh,

Bihar, Jharkhand, West Bengal and Assam are the worst affected

• Methods to tackle Arsenic contamination are oxidation, co-precipitation,

adsorption, ion exchange and membrane process, permeable reactive barriers,

phytoremediation, biological treatment and electro kinetic treatment.